Download as pdf or txt
Download as pdf or txt
You are on page 1of 67

SMA 2270: CALCULUS III

⃝Francis
c O. Ochieng
francokech@gmail.com

Department of Pure and Applied Mathematics


Jomo Kenyatta University of Agriculture and Technology

Course Description
• Limits, continuity and differentiability.

• Sequences and series: convergence tests.

• Mean Value Theorem of differential calculus. L’Hôpital’s rule. Rolle’s theorem.

• Power series: Taylor’s and Maclaurin’s theorems including applications to binomial theorem.

• Logarithmic, exponential, trigonometric, and hyperbolic functions.

• Trigonometric and hyperbolic representation of complex numbers.

• Partial differentiation: first and second partial derivatives, total derivatives, and change of
variables for two independent variables.

• Determinants: their evaluation and properties.

• Matrices: operations, inverse, solution of simultaneous linear equations. Cramer’s rule,


eigenvalues and eigenvectors. Application to geometrical transformations in two and three
dimensions.

• Integration: reduction formulae, applications to arc length, plane and surface area, volume, mass
centre and moments of inertia in Cartesian and polar co-ordinates. Improper integrals and their
convergence. Integration as the limit of a sum including pincer method for evaluation of simple
integrals. Double integrals including change of order of integration and change of variable.

References
[1] Calculus: Early Transcendentals (8th Edition) by James Stewart

[2] Calculus With Analytic Geometry (5th edition) by Larson Roland E., Hostetler Robert P., and
Bruce H. Edwards

[3] Calculus and Analytical Geometry (9th edition) by George B. Thomas and Ross L. Finney

[4] Advanced Engineering Mathematics (10th ed.) by Erwin Kreyszig

[5] Calculus by Larson Hostellem

Lecture 1

1
⃝Francis
c Oketch

1 Limits
Definition 1.1 (Basic limit definition). Let f (x) be a function and let a and L be real numbers. If
f (x) approaches L as x approaches a (but is not equal to a), then we say that f (x) has limit L as x
approaches a and we denote
. = L.
lim f (x)
x→a

→ Note: lim f (x) is the value that f (x) approaches as x approaches a, and a does not have to be in
x→a
the domain of f .

1.1 Properties of limits


Theorem 1.1. Suppose lim f (x) = L1 and lim g(x) = L2 , then
x→a x→a

1. [Addition rule] lim [f (x) + g(x)] = L1 + L2


x→a

2. [Scalar multiple] lim [λf (x)] = λL1 , where λ is a constant.


x→a

3. [Product rule] lim [f (x) · g(x)] = L1 · L2


x→a
[ ]
f (x) L1
4. [Quotient rule] lim =
x→a g(x) L2

1.2 Methods of evaluating limits of functions


 Direct substitution
The required limit is obtained by just plugging in the value of input, say x, into the given
function, say f (x).

Example(s):

(a) Evaluate lim 3x3 − x2 + 2x + 5.


x→2
Solution

lim (3x3 − x2 + 2x + 5) = 3 lim x3 − lim x2 + 2 lim x + lim 5)


x→2 x→2 x→2 x→2 x→2
= 3(2 ) − (2 ) + 2(2) + 5
3 2

= 29

x2 − 1
(b) Evaluate lim .
x→1 x + 1
Solution
x2 − 1 D.S 12 − 1 0
lim = = =0
x→1 x + 1 1+1 2

 Factorization
If on direct substitution we get the indeterminate form 0/0, then it means that there is a
common factor in both the numerator and denominator. In this case, we perform factorization
first so as to simplify the given function.

→ Note: if the polynomial in the numerator is of degree greater than the degree of the polynomial
in the denominator, we first need to perform long division.

Example(s):

2
1.2 Methods of evaluating limits of functions ⃝Francis
c Oketch

x2 + x − 6
(a) Evaluate lim
x→2 x−2
Solution
x2 + x − 6 (x − 2)(x + 3)
lim = lim
x→2 x−2 x→2 x−2
D.S
= lim (x + 3) = 2 + 3
x→2
= 5
x2 + 3x + 2
(b) Evaluate lim
x→−2 2x2 − 8
Solution
x2 + 3x + 2 (x + 2)(x + 1)
lim = lim
x→−2 2x2 − 8 2(x + 2)(x − 2)
x→−2
x + 1 D.S −2 + 1 −1
= lim = =
x→−2 2(x − 2) 2(−2 − 2) −8
1
=
8
x3 − 1
(c) Evaluate lim .
x→1 x2 − 1
Solution
( )
x3 − 1 x−1
lim 2 = lim x + 2 (long division)
x→1 x − 1 x→1 x −1
[ ]
x−1
= lim x + (factorization)
x→1 (x − 1)(x + 1)
( )
1 D.S 1 1
= lim x + = 1+ =1+
x→1 x+1 1+1 2
3
=
2
 Limits at infinity
In this case, we first divide the numerator and denominator by the highest power of x in the
denominator.

Example(s):
5x3 − 1
(a) Evaluate lim .
x→∞ 4x3 − 2x − 7

Solution
1
5x3 − 1 5−
lim = lim x3
x→∞ 4x3 − 2x − 7 x→∞ 2 7
4− 2
− 3
x x
1
5− 5−0
D.S
= ∞ =
4−
2

7 4−0−0
∞ ∞
5
=
4
 Rationalization
Suppose there exists surds in either the numerator or denominator or both. Then, we first need
to multiply both the numerator and denominator by the conjugate of the factor containing the
surd (in either the numerator or denominator) and then simplify the resulting function. After
rationalization, we perform a direct substitution.

3
1.2 Methods of evaluating limits of functions ⃝Francis
c Oketch

→ Note: in case the surds appear in both the numerator and denominator, then we rationalize
the denominator.

Example(s):

(a) Evaluate lim x2 − 4x − x.
x→∞

Solution
(√ )
√ (√ ) x2 − 4x + x
lim x2 − 4x − x = lim x2 − 4x − x √
x→∞ x→∞ x2 − 4x + x
x2 − 4x − x2 −4x
= lim √ = lim √
x→∞ x − 4x + x x→∞ x − 4x + x
2 2
1
−4x · −4
lim (√ x = lim √
= ) 1
x→∞
x2 − 4x + x ·
x→∞ 4
1− +1
x x
D.S −4 −4 −4
= √ =√ =
4 1−0+1 1+1
1− +1

= −2

x−3
(b) Evaluate lim .
x→9 x − 9

Solution
√ √ √
x−3 ( x − 3)( x + 3)
lim = lim √
x→9 x − 9 x→9 (x − 9)( x + 3)
(x − 9) 1
= lim √ = lim √
x→9 (x − 9)( x + 3) x→9 x+3
D.S 1 1
= √ =
9+3 3+3
1
=
6

Exercise:
√ √
(a) lim x2 − 2 − x2 + x.
x→∞

Solution
(√
√ )

√ √ √ x2 − 2 + x2 + x
lim x2 −2− x2 +x = lim x2−2− +x · √
x2 √
x→∞ x→∞ x2 − 2 + x2 + x
(x2 − 2) − (x2 + x) −2 − x
= lim √ √ = lim √ √
x→∞ x2 − 2 + x2 + x x→∞ x2 − 2 + x2 + x
2 2
− −1 − −1
= lim √ x √ D.S
= √ ∞ √
x→∞ 2 1 2 1
1− 2 + 1+ 1− + 1+
x x ∞ ∞
1
= −
2

x3 − 1
(b) Evaluate lim . [ans: 3]
x→1 x − 1

1− x
(c) Evaluate lim . [ans: 1/2]
x→1 1 − x

4
1.3 One-Sided Limit ⃝Francis
c Oketch

5x2 − 3x + 2
(d) Evaluate lim . [ans: 1/2]
x→∞ 10x2 − x + 100
(e) Evaluate the following

1+ x
i) lim √ . [ans: 1/2]
x→0 2 + 2x
x
ii) lim √ . [ans: 2]
x→0 1 − 1 − x

→ Note: A function which grows arbitrarily large as x goes to positive or negative infinity is said to
have an infinite limit. Infinity is not a real number, so if a function has infinite limit, we
say that the limit does not exist.

1.3 One-Sided Limit


Definition 1.2 (Right-Hand Limit). If a function f (x) approaches the number L as x approaches the
real number a and x > a, then we say that L is the right-hand limit of f at x = a and write

lim f (x)
. = L.
x→a+

Definition 1.3 (Left-Hand Limit). If a function f (x) approaches the number L as x approaches the
real number a and x < a, then we say that L is the left-hand limit of f at x = a and write

lim f (x)
. = L.
x→a−

→ Note: the limit of f (x) as x approaches a exists if both left and right limits exist and are equal at
x = a. In that case, we have

lim f (x) = lim f.(x) = lim f (x) = L


x→a− x→a+ x→a

Example(s):



3
x ,
if x < 1
(a) Consider the function defined by f (x) = 1, if x = 1 . Evaluate lim f (x).

 x→1
2 − x, if x > 1

Solution

(i) LHL: lim f (x) = lim (x3 ) = 13 = 1


x→1− x→1−
(ii) RHL: lim f (x) = lim (2 − x) = 2 − 1 = 1
x→1+ x→1+
(iii) Since the result (i) = (ii), we get lim f (x) = 1
x→1

|5x|
(b) Evaluate lim .
x→0 x

Solution
{
−5x
|5x| x if x < 0
Here, f (x) = = 5x
. Now,
x x if x > 0

|5x| −5x
(i) LHL: lim = lim = −5
x→0− x x→0 − x
|5x| 5x
(ii) RHL: lim = lim =5
x→0 + x x→0 x
+

|5x|
(iii) Since (i) ̸= (ii), therefore, lim does not exist.
x→1 x

5
1.4 L’Hôpital’s rule ⃝Francis
c Oketch

→ Note (absolutely stated functions): to separate the function contained in the absolutely stated
function, first identify the reference point by equating the interior term to 0 and then investigate signs
of the interior expression to LHS and RHS of the reference point. For example,
 If f (x) = |x − 3|. The reference point is x − 3 = 0 ⇒ x = 3. Thus,
{
−(x − 3) if x < 3
f (x) =
+(x − 3) if x ≥ 3

 If f (x) = 5 + |x + 5|. The reference point is x + 5 = 0 ⇒ x = −5. Thus,


{
5 − (x + 5) if x < −5
f (x) =
5 + (x + 5) if x ≥ −5

 1

 if x < 0

2 + x
1
 If f (x) = . The reference point is x = 0. Thus, f (x) = .
2 − |x| 

  1
 if x ≥ 0
2−x
Exercise:


 x − 2x
2 if x < 1
(a) Consider the function defined by f (x) = 2 if x = 1 . Evaluate lim f (x) and

 3x − 4 x→1−
if x > 1.
lim f (x).
x→1+
{
2 − 3x if x ≤ 1
(b) Consider the function defined by f (x) = . Does lim f (x) exist?
2x3 if x > 1 x→1
{
x if x ̸= 0
(c) Find the value of lim f (x) where f (x) =
x→0 1 if x = 0.

1.4 L’Hôpital’s rule


f (x) 0 f (x) ±∞
If on direct substitution we get indeterminate form −i.e., either lim = or lim = .
x→a g(x) 0 x→a g(x) ±∞
Then,
f (x) . f ′ (x)
lim = lim ′
x→a g(x) x→a g (x)

provided the limit exists. Repeat finding derivatives until you get a meaningful result.

→ Note: if on direct substitution we get forms other than 0/0 or ±∞/ ± ∞ (i.e., 00 , 0±∞ , 1±∞ ,
±∞±∞ ), then we first transform the problem either by introducing the natural logarithm (ln) or by
1
writing them in the simplest form e.g cosec x = .
sin x
Example(s):
x2 + x
(a) Find lim .
x→∞ e2x + 1

Solution
x2 + x ∞
On direct substitution, we have lim = (indeterminate). So,
2x
x→∞ e + 1 ∞
x2 + x L′ H 2x + 1 D.S ∞
lim = lim = (indeterminate)
x→∞ e2x + 1 x→∞ 2e2x ∞
L′ H 2 D.S 2
= lim =
x→∞ 4e2x ∞
= 0

6
1.4 L’Hôpital’s rule ⃝Francis
c Oketch

x − sin x
(b) Evaluate lim
x→0 x3

Solution
x − sin x 0 − sin 0 0
On direct substitution, we have lim 3
= 3
= (indeterminate). So,
x→0 x 0 0
x − sin x L′ H 1 − cos x D.S 0
lim = = (indeterminate)
x→0 x3 3x2 0
L′ H sin x D.S 0
= lim = (indeterminate)
x→0 6x 0
L′ H cos x
= lim
x→0 6
D.S 1
=
6
( )x/2
(c) Evaluate lim 2x
x→0+

Solution
On direct substitution, we have lim (2x)x/2 = 00 (indeterminate). So,
x→0+

Let y = (2x)x/2
x 1 ln(2x)
⇒ ln(y) = ln(2x) =
2 2 1/x
1 ln(2x) D.S ∞
⇒ lim ln(y) = lim = (indeterminate)
x→0+ 2 x→0+ 1/x ∞
2
L′ H 1 2x 1 D.S
= lim −1 = lim (−x) = 0
2 x→0+ x2
2 x→0+
[ ]
∴ ln lim y = 0 ⇒ lim y = e0 = 1
x→0+ x→0+
( )x
1
(d) Evaluate lim 1+ .
x→∞ x

Solution
( )x
1
On direct substitution, we have lim 1+ = 1∞ (indeterminate). So,
x→∞ x
( )x
1
Let y = 1+
x
( )
1
( )
ln 1 +
1 x
⇒ ln(y) = x ln 1 + =
x 1/x
( )
1
ln 1 +
x D.S 0
⇒ lim ln(y) = lim = (indeterminate)
x→∞ x→∞ 1/x 0
1 −1
( )
1 · x2
L′ H 1+ x 1 D.S
= lim = lim = 1
x→∞ −1 x→∞ 1 + 1
x
x2
[ ]
∴ ln lim y = 1 ⇒ lim y = e1 = e
x→∞ x→∞

Exercise:

7
1.4 L’Hôpital’s rule ⃝Francis
c Oketch

1. Evaluate
2 sin x − sin 2x
(a) lim
x→0 2ex − 2 − x2
7x − 28
(b) lim
x→∞ x3
x2 − x − 2
(c) lim
x→1 x2 − 1
2. Evaluate
1
(a) lim x x−1 . [ans: = e]
x→1
(b) lim xsin x . [ans: = 1]
x→0
(c) lim xx . [ans: = 1]
x→0

3. Evaluate
sin 7x
(a) lim . [ans: = 7/4]
x→0 4x
cot x
(b) lim .
x→0 ln x2
+
( )
cos y
(c) limπ . [ans: = 1]
2 −y
π
y→ 2

1 − cos θ
(d) lim . [ans: = 0]
θ→0 θ
sin2 θ
(e) lim . [ans: = 1]
θ→0 θ 2

4. In each case, evaluate the limit.

(a) lim xa ln x, where a > 0.


x→0+
( ( ))2x
2
(b) lim 1 + sin . [ans: = e4 ]
x→∞ x
( )
2 2
(c) lim −
x→0+ x sin x

(d) lim cos xcot x


x→0+

(e) lim x2 + 1 − x
x→∞

5. Amy knows the following values for f (x) and g(x): f (−6) = 1, g(−6) = 1, f ′ (−6) = 6 and
g ′ (−6) = −5. Assume all functions are continuous, find the following.
ln(g(x)) 5
(a) lim . [ans: = − ]
x→−6 [f (x)]2 − 1 12
[g(x) − 1]2 25
(b) lim . [ans: = ]
x→−6 [f (x) − 1]2 36
g(x)[f (x) − 1] 6
(c) lim . [ans: = − ]
x→−6 f (x)[g(x) − 1] 5

Lecture 2

8
1.5 The ϵ − δ definition of limits ⃝Francis
c Oketch

1.5 The ϵ − δ definition of limits


The statement lim f (x) = L means that given any real number ϵ > 0, there exists another real number
x→a
δ > 0 such that
0 < |x − a| < δ ⇒ . |f (x) − L| < ϵ
Diagrammatically, we have:

The value of δ will depend on the value of ϵ i.e we will always begin with any ϵ > 0 and then determine
an appropriate corresponding value of δ > 0. If there is a single ϵ > 0 for which this process fails, then
the limit L has been incorrectly computed or the limit does not exist.

Example(s):

1. Show that lim (2 − 3x) = 5 using the ϵ − δ definition of limit.


x→−1

Solution
Let ϵ > 0 be given. We need to find δ > 0 (which depends on ϵ) so that if 0 < |x + 1| <
δ, then |(2 − 3x) − 5| < ϵ. We begin with |(2 − 3x) − 5| < ϵ and ”solve for” |x + 1|. So,

0 < |x + 1| < δ ⇒ |(2 − 3x) − 5| < ϵ


⇒ | − 3x − 3| < ϵ
⇒ | − 3(x + 1)| < ϵ
⇒ | − 3||x + 1| < ϵ
ϵ
∴ |x + 1| <
3
ϵ ϵ
Now, choose δ = . Thus, if 0 < |x + 1| < , it follows that |(2 − 3x) − 5| < ϵ. This completes
3 3
the proof.

2. Show that lim (x2 + 3) = 4 using the ϵ − δ definition of limit.


x→1

Solution
Let ϵ > 0 be given. We need to find δ > 0 (which depends on ϵ) so that if 0 < |x − 1| <
δ, then |(x2 + 3) − 4| < ϵ. We begin with |(x2 + 3) − 4| < ϵ and ”solve for” |x − 1|. So,

0 < |x − 1| < δ ⇒ |(x2 + 3) − 4| < ϵ


⇒ |x2 − 1| < ϵ
⇒ |(x − 1)(x + 1)| < ϵ
⇒ |x − 1||x + 1| < ϵ

We replace |x + 1| with an appropriate constant by arbitrarily assuming that δ ≤ 1 (this


assumption is valid since, in general, once we find a δ that works, all smaller values of δ also
work). Then,
|x − 1| < δ ≤ 1 ⇒ −1 < x − 1 < 1 ⇒ 0 < x < 2

9
1.5 The ϵ − δ definition of limits ⃝Francis
c Oketch

Clearly, the function f (x) = x2 + 3 is defined for all x in the interval (0,2). Hence, this range of
values of x is appropriate. Therefore,

0<x<2 ⇒ 1<x+1<3

It follows that
ϵ
|x − 1||x + 1| < |x − 1|(3) < ϵ ⇒ |x − 1| <
3
( )
ϵ
Now, choose δ = min 1, (This guarantees that both assumptions made about δ in the course
3
of this proof are taken into account simultaneously.). This completes the proof.

3. Show that lim (3x2 − x) = 10 using the ϵ − δ definition of limit.


x→2

Solution
Let ϵ > 0 be given. We need to find δ > 0 (which depends on ϵ) so that if 0 < |x − 2| <
δ, then |(3x2 − x) − 10| < ϵ. We begin with |(3x2 − x) − 10| < ϵ and ”solve for” |x − 2|. So,

0 < |x − 2| < δ ⇒ |(3x2 − x) − 10| < ϵ


⇒ |(x − 2)(3x + 5)| < ϵ
⇒ |x − 2||3x + 5| < ϵ

We replace |3x + 5| with an appropriate constant by arbitrarily assuming that δ ≤ 1. Then,

|x − 2| < δ ≤ 1 ⇒ −1 < x − 2 < 1 ⇒ 1<x<3

Clearly, the function f (x) = 3x2 − x is defined for all x in the interval (1,3). Hence, this range
of values of x is appropriate. Therefore,

1<x<3 ⇒ 3 < 3x < 9 ⇒ 8 < 3x + 5 < 14

It follows that
ϵ
|x − 2||3x + 5| < |x − 2|(14) < ϵ ⇒ |x − 2| <
14
( )
ϵ
Now, choose δ = min 1, . This completes the proof.
14
( )
2 1
4. Show that lim = using the ϵ − δ definition of limit.
x→3 x+3 3

Solution
Let ϵ > 0 be given. depends on ϵ) so that if 0 < |x − 3| <
We need to find δ > 0 (which
2 1 2 1
δ, then − < ϵ. We begin with − < ϵ and ”solve for” |x − 3|. So,
x+3 3 x+3 3

2 1
0 < |x − 3| < δ ⇒ − <ϵ
x + 3 3

6 − (x + 3)
⇒ <ϵ
3(x + 3)

−1(x − 3)
⇒ <ϵ
3(x + 3)
| − 1||x − 3|
⇒ <ϵ
|3||x + 3|
1 |x − 3|
⇒ <ϵ
3 |x + 3|
1
⇒ |x − 3| < 3ϵ
|x + 3|

10
1.5 The ϵ − δ definition of limits ⃝Francis
c Oketch

1
We replace with an appropriate constant by arbitrarily assuming that δ ≤ 1. Then,
|x + 3|

|x − 3| < δ ≤ 1 ⇒ −1 < x − 3 < 1 ⇒ 2<x<4


2
Clearly, the function f (x) = is defined for all x in the interval (2,4). Hence, this range of
x+3
values of x is appropriate. Therefore,
1 1 1
2<x<4 ⇒ 5<x+3<7 ⇒ < <
7 x+3 5
It follows that ( )
1 1
|x − 3| < |x − 3| < 3ϵ ⇒ |x − 3| < 15ϵ
|x + 3| 5
Now, choose δ = min (1, 15ϵ). This completes the proof.
( )
x+3
5. Show that lim √ = 2 using the ϵ − δ definition of limit.
x→1 1+ x

Solution
Let ϵ > 0 be given. We need to find δ > 0 (which depends on ϵ) so that if 0 < |x − 1| <
x+3 x+3
δ, then √ − 2 < ϵ. We begin with √ − 2 < ϵ and ”solve for” |x − 1|. So,
1+ x 1+ x

x+3
0 < |x − 1| < δ
⇒ √ − 2 < ϵ
1+ x

x + 3 − 2(1 + √x)

⇒ √ <ϵ
1+ x

x + 1 − 2 x

⇒ √ <ϵ
1+ x

We need to be able to factor out (x − 1) from the numerator.



x + 1 − 1 + 1 − 2√x

⇒ √ <ϵ
1+ x

(x − 1) + 2(1 − x)

⇒ √ <ϵ
1+ x

(x − 1)(1 + x) + 2(1 − √x)(1 + √x)


⇒ √ √ <ϵ
(1 + x)(1 + x)

(x − 1)(1 + x) + 2(1 − x)

⇒ √ 2 <ϵ
(1 + x)

(1 + x − 2)

⇒ (x − 1) √ <ϵ
(1 + x)2
√ 1
⇒ |x − 1| x − 1 √ <ϵ
(1 + x)2
√ 1
We replace | x − 1| and √ with appropriate constants by arbitrarily assuming that
(1 + x)2
δ ≤ 1. Then,
√ √
|x − 1| < δ ≤ 1 ⇒ −1 < x − 1 < 1 ⇒ 0<x<2 ⇒ 0< x< 2
x+3 √ √
Clearly, the function f (x) = √ is defined for all x in the interval (0, 2). Hence, this
√ 1+ x
range of values of x is appropriate. Therefore,
√ √ √ √ √
i) 0 < x < 2 ⇒ −1 < x − 1 < 2 − 1 ⇒ −1 < x − 1 < 1.

11
⃝Francis
c Oketch

√ √ √ √ √ 2 √
ii) 0 < x < 2 ⇒ 1 < 1+ x < 2+1 ⇒ 12 < (1 + x) < ( 2 + 1)2 ⇒
1 1
√ < √ 2 < 1.
( 2 + 1) 2 (1 + x)
It follows that
√ 1
|x − 1| x − 1 √ < |x − 1|(1)(1) < ϵ ⇒ |x − 1| < ϵ
(1 + x)2

Now, choose δ = min (1, ϵ). This completes the proof.

Exercise:

1. Using the ϵ − δ definition of limit of a function, show that;


( )
18x2 − 3x − 1 ϵ
(i) lim = 3. [ans: δ = ]
x→ 13 3x − 1 6
( ) √
(ii) lim x2 − 8x + 8 = −8. [ans: δ = ϵ]
x→4
( ) ( )
2 ϵ
(iii) lim x + 1 = 65. [ans: δ = min 1, ]
x→8 17
( ) ( )
x+4 1 28
(iv) lim =− . [ans: δ = min 1, ϵ ]
x→−6 2 − x 4 3
( ) ( )
x 1 1 2
(v) lim = 1. [hint: assume that δ ≤ , ans: δ = min , ϵ ]
x→3 4x − 9 4 4 3
( √ ) { √ }
(vi) lim 2 + x = 5. [ans: δ = min 1, ( 8 + 3)ϵ ]
x→9
(√ ) { √ }
(vii) lim x + 5 = 3. [ans: δ = min 1, ( 8 + 3)ϵ ]
x→4

2. Use the precise definition of limits to show that:

x2 + x − 6
(i) lim = −5
x→−3 x+3
(ii) lim (4x3 + 3x2 − 24x + 22) = 5
x→1
(iii) lim (−2x3 + 9x + 4) = −3
x→−1
(iv) lim (2 + 5x) = 17.
x→3

2 Continuity and Differentiability


2.1 Continuity
• [Continuity at a Point] A function f (x) is said to be continuous at an interior point x = a if the
following three conditions are satisfied:

(i) f (a) is defined (finite)


(ii) lim f (x) exists (i.e LHL=RHL)
x→a
(iii) lim f (x) = f (a)
x→a

→ Note: if at least one of these conditions is not satisfied, then f (x) is not continuous at x = a.
In this case, we say that f (x) is discontinuous at point a or that the point a is a discontinuity
of f .

• [Continuity on an Interval] A function f is said to be continuous on an interval (a, b) if it is


continuous for every c in (a, b).

12
2.1 Continuity ⃝Francis
c Oketch

Example(s):
 2

 x −1
 if x < −1
(a) Discuss the continuity of the function f (x) = x+1 at x = −1



x2 − 3 if x ≥ −1

Solution
We need to test the three conditions for continuity:

(i) f (−1) = (−1)2 − 3 = −2 (defined).


(ii) lim f (x):
x→−1

( )
x2 − 1 (x  − 1)
+1)(x

LHL: lim f (x) = lim = lim  = lim (x − 1) = −2
x→−1− x→−1− x+1 x→−1− (x
 +1) x→−1−

RHL: lim f (x) = lim (x2 − 3) = 1 − 3 = −2


x→−1+ x→−1+

Since LHL=RHL=-2, therefore, lim f (x) = −2


x→−1
(iii) So, as lim f (x) = f (−1) therefore, f (x) is continuous on (−4, 4).
x→−1

2x4 − 6x3 + x2 + 3
(b) Discuss the continuity of the function f (x) = at x = 1.
x−1

Solution
2x4 − 6x3 + x2 + 3
Clearly, the function f (x) = is discontinuous at x = 1. However, the
x−1
discontinuity can be removed by first simplifying the given function. Thus, by long division we
have
2x3 − 4x2 − 3x − 3
)
x−1 2x4 − 6x3 + x2 +3
− 2x4 + 2x3
− 4x3 + x2
4x3 − 4x2
− 3x2
3x2 − 3x
− 3x + 3
3x − 3
0
Hence, the function can be rewritten in the simplified form f (x) = 2x3 − 4x2 − 3x − 3 [by long
division], which is now continuous at x = 1 [student to verify this]. Therefore, the original
function is said to have a removable discontinuity.

Exercise:

(a) Find the value of A and B so that the following function is continuous for all x.
 ( )

 1 − cos x

 A if x < 0

 sin2 x




f (x) = 2x2 − x + B if 0 ≤ x ≤ 1







 x2 + 2x − 3

 if x > 1
2 x −1

13
2.2 Differentiability ⃝Francis
c Oketch

Solution

f (0) = 2(0)2 − (0) + B = B


A(1 − cos(x)) A( −(
(1( (((
cos(x)) A
lim f (x) = lim = lim ( ( ( =
x→0− x→0− 2
sin (x) x→0− ( −(
(1( cos(x))(1 + cos(x)) 2
lim f (x) = lim (2x − x + B) = B
2
x→0+ x→0+

Since f (x) is continuous at x = 0, it follows that lim f (x) = lim f (x) = f (0). Thus, we have
x→0− x→0+
A
= B − − − (∗).
2

Also,

f (1) = 2(1)2 − (1) + B = 1 + B


lim f (x) = lim (2x2 − x + B) = 1 + B
x→1− x→1−
x2 + 2x − 3 (x  + 3)
−1)(x
 4
lim f (x) = lim = lim  = =2
x→1+ x→1+ x −1
2 x→1 
+ − 1)(x + 1)
(x 2
Since f (x) is continuous at x = 1, it follows that lim f (x) = lim f (x) = f (1). Thus, we have
x→1− x→1+
1 + B = 2 − − − (∗∗).

Solving equations (∗) and (∗∗) simultaneously yields A = 2 and B = 1.



 3
 x + 27 , if x ̸= −3
(b) Discuss the continuity of the function f (x) = x+3 .

27, if x = −3

(c) Find a and b so that the following function is continuous for all x in the real number line:


2, if x < 1

f (x) = ax + b, if 1 ≤ x ≤ 2


6, if x > 2

[ans: a = 4, b = −2]

2.2 Differentiability
When the limit of a function f (x) at point x = a exists, then from the first principle the derivative of
f (x) at x = a is denoted by f ′ (a) and is defined by

f ′ (a) = lim . + h) − f (a)


f (a
h→0 h

The right-hand derivative of f (x) at x = a is defined by

f (a + h) − f (a)
Rf ′ (a) = lim .
h→0 + h

The left-hand derivative of f (x) at x = a is defined by

f (a + h) − f (a)
Lf ′ (a) = lim .
h→0 − h

→ Note: if Rf ′ (a) = Lf ′ (a) at a point x = a, then we say that f (x) is differentiable at x = a. Hence,
the test for differentiability is simply to compute Rf ′ (a) and Lf ′ (a) and compare the results.

Example(s):

14
2.3 Rolle’s Theorem ⃝Francis
c Oketch

1. Show that f (x) = |x| is differentiable everywhere except at x = 0 and illustrate the solution by
a graph.

Solution


−x,
 if x < 0
f (x) = |x| =


x, if x ≥ 0
So,
f (0 + h) − f (0) 0+h−0
Rf ′ (0) = lim = lim =1
h→0+ h h→0 + h
f (0 + h) − f (0) −0 − h + 0
Lf ′ (0) = lim = lim = −1
h→0 − h h→0 − h
But Rf ′ (0) ̸= Lf ′ (0), so f (x) is not differentiable at x = 0. Draw the graph of y = −x and
y = x on the same set of axes.

Exercise:
1. Show that f (x) = x2 is differentiable at x = 1.

2. Although differentiability implies continuity, continuity does not imply differentiability. Discuss
using appropriate example. [hint: use f (x) = |x − 2|. Test it for both continuity and
differentiability.]

2.3 Rolle’s Theorem


Suppose that a function f (x) is continuous on the closed interval [a, b] and is differentiable on the
open interval (a, b). If f (a) = f (b), then there exists a real number c between a and b (i.e., a < c < b)
such that
f ′ (c). = 0 .

→ Note: In other words, since f (x) is continuous


and differentiable, then in between every two
points a and b with the same image, the function
f (x) must have a maximum or a minimum.

Example(s):
1. Find the value of c ∈ [−3, 0] as prescribed by Rolle’s theorem if f (x) = (x2 + 3x)e−x/2 .

Solution
Clearly, the function f (x) is continuous on the closed interval [−3, 0] and differentiable on the
open interval (−3, 0). Here, a = −3 and b = 0. Then,

f (a) = f (−3) = ((−3)2 + 3(−3))e−(−3)/2 = 0


f (b) = f (0) = ((0)2 + 3(0))e−(0)/2 = 0
1 1
f ′ (x) = (x2 + 3x)(− e−x/2 ) + (2x + 3)e−x/2 ⇒ f ′ (c) = (c2 + 3c)(− e−c/2 ) + (2c + 3)e−c/2
2 2
Since f (−3) = f (0) = 0, then by Rolle’s theorem there must be a point c ∈ [−3, −] such that
1
f ′ (c) = 0. So, (c2 + 3c)(− e−c/2 ) + (2c + 3)e−c/2 = 0 which yields c = −2 or c = 3. But only
2
−2 lies between −3 and 0. Therefore, c = −2.

15
2.4 Mean Value Theorem (MVT) ⃝Francis
c Oketch

2.4 Mean Value Theorem (MVT)


Suppose that the function f (x) is continuous on the closed interval [a, b], and is differentiable on the
open interval (a, b). Then, there exists a real number c between a and b such that

f ′ (c) = . − f (a)
f (b)
b−a

→ Note: In other words, since f (x) is continuous


and differentiable, then in between every two
points a and b, there is a point c for which the
slope of the tangent line at c is equal to the slope
of the secant line linking the points (a, f (a)) and
(b, f (b)).

Example(s):
1. Verify the MVT for the function f (x) = 2x2 − 7x + 10 in the closed interval [2, 5].

Solution
Clearly, the function f (x) is continuous on the closed interval [2, 5] and differentiable on the
open interval (2, 5). Here, a = 2 and b = 5. Then,
f (a) = f (2) = 2(2)2 − 7(2) + 10 = 4
f (b) = f (5) = 2(5)2 − 7(5) + 10 = 25
f ′ (x) = 4x − 7 ⇒ f ′ (c) = 4c − 7
f (b) − f (a) 25 − 4
So by MVT, we have f ′ (c) = ⇒ 4c − 7 = which yields c = 3.5. Since
b−a 5−2
c = 3.5 is strictly in the interval [2, 5], it follows that MVT holds (or is satisfied).
2. Given that f (x) is continuous and differentiable on [-6,-1], f (−6) = −23, and f ′ (x) ≥ 4. Find
the smallest possible value of f (−1). [ans: 3 ≤ f (−1)]
3. Verify that the function f (x) = sin x satisfies the hypothesis of MVT on the interval [0, π/2].
Then find all c that satisfy the conclusion of the MVT. [ans:c = 0.4965π]
4. For the function g(x) = (x + 1)3 on [−1, 1], find the number c guaranteed by the Mean Value
Theorem.
x−4
5. For f (x) = , decide if we can use the Mean Value Theorem for derivatives on [0, 5] or [4, 6].
x−3
If so, find c. If not, explain why?

Lecture 3

3 Sequences and Series


3.1 Sequences
A sequence is an arrangement of terms (either numbers or functions) in a definite order with a rule of
generating its members. It is also a function whose domain is the set of natural numbers. E.g.,
(a) 1, 3, 5, 7, · · · The nth term is an = 2n − 1.
(b) 1, 4, 9, 16, · · · The nth term is an = n2 .
(c) log x, log x2 , log x3 , · · · The nth term is an = log xn .

16
3.2 Series ⃝Francis
c Oketch

3.1.1 Limit of a sequence


The sequence {an }n≥1 is said to approach the number L as a limit when n → ∞ provided that there
exists a real number ϵ > 0 and an integer N such that |an − L| < ϵ whenever n ≥ N . If so, then

lim a.n = L
n→∞

When this condition is satisfied, then the sequence {an } converges to the limit L. Any sequence which
is not convergent is said to be divergent.

Example(s):
n2
1. Does the sequence {an }n≥1 = converge as n → ∞?
n2 + 3
Solution
n2 D.S ∞
lim an = lim = (indeterminate)
n→∞ n→∞ n2 + 3 ∞
L′ H 2n D.S ∞
= lim = (indeterminate)
n→∞ 2n ∞
L′ H 2
= lim
n→∞ 2
= 1
n2
Since the limit exists i.e., lim = 1, the sequence {an } is convergent.
n→∞ n2 + 3

2. Investigate the convergence of the following sequences as n → ∞.


1 + cos n
(a) {an } =
n
3n4 + n − 1
(b) {an } = 4
5n + 2n2 + 1
n
(c) {an } = 2
2n + n + 1
(d) {an } = (−1)n

(e) {an } = n2 + 4n − n [ans: L = 2]

3.2 Series
The sum of the terms in a sequence forms a series. Normally, we use Sigma (Σ) to denote summation.
n+1
For example, consider the sequence {an }n≥1 = ln( ). The series is given by
n

∑ ∞
∑ n+1
an = ln( )
n=1 n=1
n

3.2.1 Special series


(1) Geometric series
This is a series of the form ∞

ark , with a ̸= 0
k=0

The series

(i) Converges if |r| < 1


(ii) Diverges if |r| > 1

17
3.2 Series ⃝Francis
c Oketch

Without proof, the sum of the first n terms of a GP is given by




 a(rn − 1)
 , if |r| > 1
Sn = r−1

 a(1 − rn )
 , if |r| < 1
1−r

(2) P-series
This is a series of the form ∞
∑ 1
,
k=0
kp
where p is a positive constant. The p-series

(i) Converges if p > 1, and


(ii) Diverges if 0 < p ≤ 1.

→ Note: this will be confirmed by the integral test.

(3) Harmonic series


This is a series of the form ∞
∑ 1
k=0
k
Thus, harmonic series is a special type of p-series, obtained by setting p = 1.

→ Note: harmonic series always diverges (to be confirmed by the integral test).

3.2.2 Limits and convergence of series



∑ ∑
n
Let ak − − − (∗) be a series and Sn = ak denote the nth partial sum of the infinite series (∗).
k=1 k=1
Thus, we have

S1 = a1
S2 = a1 + a2
S3 = a1 + a2 + a3
.. .
. = ..
Sn = a1 + a2 + a3 + · · · + an

The partial sums form a sequence denoted by {Sn }. The infinite series converges to the sum S if

lim S.n = S
n→∞

The infinite series diverges if the sequence {Sn } does not converge. We call S the sum of the series.

→ Note: if a limit exists, it must be unique.

Example(s):

∑ 1
1. Show that the series converges and find its sum.
k=1
2k

Solution

∑ 1 1 1 1 1
(i) The series is = + 2 + 3 + 4 + · · · is a geometric series (GP).
k=1
2k 2 2 2 2

18
3.2 Series ⃝Francis
c Oketch

(ii) Partial sums:


1
S1 =
2
1 1 3
S2 = + 2 =
2 2 4
1 1 1 7
S3 = + 2+ 3 =
2 2 2 8
1 1 1 1 15
S4 = + + + =
2 22 23 24 16
.. ..
. = .
Sn = ?
1 3 7 15
(iii) Sequence of partial sums: , , , , · · · The nth term is given by
2 4 8 16
( )
1
2 1 − ( 12 )n 1
Sn = =1−
1− 1
2
2n
( 1)
Now, lim Sn = lim 1 − n = 1. Therefore, the given infinite series converges since Sn
n→∞ n→∞ 2
converges and its sum is 1. That is,
∑∞
1
=1
k=1
2k


2. Show that the series (−1)k diverges.
k=1

Solution


(i) The series is (−1)k = −1 + 1 − 1 + 1 − 1 + · · ·
k=1
(ii) Partial sums:

S1 = −1
S2 = −1 + 1 = 0
S3 = −1 + 1 − 1 = −1
S4 = −1 + 1 − 1 + 1 = 0
.. .
. = ..
Sn = ?

(iii) Sequence of partial sums: −1, 0, −1, 0, · · · The nth term is given by
{
−1, if n is odd
Sn =
0, if n is even

Because Sn has no limits, the given infinite series diverges.


3. Use partial fractions to find the sum of each series.

∑ 1
(a) (telescopic series)
k=1
k(k + 1)
Solution
We look for a pattern in the sequence of partial sums that might lead to a formula for Sn .
The key observation is partial fraction decomposition as follows.
1 A B
= + ⇒ A(k + 1) + B(k) = 1 ⇒ A = 1, B = −1
k(k + 1) k (k + 1)

19
3.2 Series ⃝Francis
c Oketch

∞ ∞
( )
∑ 1 ∑ 1 1
Therefore, = − and the nth partial sum is
k=1
k(k + 1) k=1 k k + 1
( ) ( ) ( ) ( ) ( )

n
1 1 1 1 1 1 1 1 1
Sn = − = 1− + − + − + ··· + −
k=1
k k+1 2 2 3 3 4 n n+1

1
Removing parentheses and canceling adjacent terms of opposite sign yields Sn = 1 −
n+1
( )
1
lim Sn = lim 1− =1
n→∞ n→∞ n+1
Therefore, the series converges and its sum is 1. So,

∑ 1
=1
k=1
k(k + 1)


∑ 4
(b) .
k=1
(4k − 3)(4k + 1)

∑ 40k
(c) .
k=1
(2k − 1)2 (2k + 1)2

4. Determine the nth partial sum of the following series. Hence, test for its convergence or
divergence.

∑ 5 1 1 1
(a) . [ans: Sn = − , S∞ = , converges]
n=1
(5n − 3)(5n + 2) 2 5n + 2 2

∑ 2n + 1 1
(b) 2 (n + 1)2
. [ans: Sn = 1 − , S∞ = 1, converges]
n=1
n (n + 1)2

5. Show whether the following series diverge or converge.


1 1 1 n
(a) + + + ··· [ans: Sn = , series converges.]
(1)(2) (2)(3) (3)(4) n+1
1 1 1 1 1 − 51n
(b) + 2 + 3 + 4 + ··· [ans: Sn = , series converges.]
5 5 5 5 4
1 1 1
(c) −1 + − + + · · ·
3 5 7
6. In the following find the formula for the nth partial sum of each series and use it to find the
series sum if the series converges.
2 2 2 (
1)
(a) 2 + + + + ··· [ans: Sn = 3 1 − , series sum = 3.]
3 9 27 3n
1 − (−2)n
(b) 1 − 2 + 4 − 8 + · · · [ans: Sn = , series diverges.]
3
7. Write down the first few terms of each series to show how the series starts. Then find the sum
of each series.

∑ (−1)k
(a)
k=0
4k
∑∞
7
(b) k
k=1
4
∞ (
∑ 5 1)
(c) +
k=0
2k 3k

20
3.2 Series ⃝Francis
c Oketch

∞ (
∑ 1 (−1)k )
(d) +
k=0
2k 5k

Lecture 4

3.2.3 Tests for convergence of series


Generally, finding a formula for the general term in the sequence of partial sums is a very difficult
process. We won’t be doing much with the partial sums of series due to the extreme difficulty faced
in finding the general formula. This also means that we’ll not be doing much work with the sum of
series since in order to get the sum we’ll also need to know the general formula for the partial sums.

(1) Ratio test


∑ a
n+1
Let an be an infinite series with nonzero terms and suppose that lim = L. Then,
n→∞ an
(i) the series converges if L < 1
(ii) the series diverges if L > 1
(iii) the test is inconclusive if L = 1
→ Note: the ratio test is often effective when the terms of a series contains factorials of
expressions involving n (e.g., n!) or expressions raised to a power involving n (e.g., np ).

Example(s):
Investigate the convergence of the following series.

∑ (−1)n 2n
(a)
n=0
n!
Solution
(−1)n 2n (−1)n+1 2n+1 (−1)(−1)n 2(2n )
Here, an = ⇒ an+1 = = . By ratio test, we
n! (n + 1)! (n + 1)n!
have
a n!
(−1)(−1)n 2(2n )
n+1
lim = lim ·

n→∞ an n→∞ (n + 1)n! (−1)n 2n
−2 −2/n

= lim = lim
n→∞ n + 1 n→∞ 1 + 1/n
0

= =0
1+0
Since L = 0 < 1, the given series converges by ratio test.
∑∞
(−1)n 2n+1
(b)
n=0
n2
Solution
(−1)n 2n+1 (−1)n+1 2n+2 (−1)(−1)n 2(2n+1 )
Here, an = ⇒ an+1 = = . By ratio test, we
n2 (n + 1)2 (n + 1)2
have
a (−1)(−1)n 2(2n+1 )n2
n+1
lim = lim ·
n→∞ an n→∞ (n + 1)2 (−1)n 2n+1
−2n2 −2n2

= lim = lim
n→∞ (n + 1)2 n→∞ n2 + 2n + 1
−2

= lim
n→∞ 1 + 2/n + 1/n2
−2

= =2
1+0+0

21
3.2 Series ⃝Francis
c Oketch

Since L = 2 > 1, the series diverges.



∑ 2n + 5
(c)
n=0
3n
∑∞
4n n!n!
(d)
n=0
(2n)!

∑ 3n
(e)
n=1
2n + 5

∑ (2n)!
(f)
n=1
(n!)3

(2) Root test


∑ √
Let an be an infinite series with nonzero terms and suppose that lim n
|an | = L. Then,
n→∞

(i) the series converges if L < 1


(ii) the series diverges if L > 1
(iii) the test is inconclusive if L = 1

Example(s):
Investigate the convergence of the following series.
∞ 2n
∑ e
(a)
n=1
nn
Solution
( )
e2n √ e2n 1/n e2

Here, an = n ⇒ |an | = n
n
= . By root test, we have
n n n

√ e2 D.S e2
lim n
an = lim = =0
n→∞ n→∞ n ∞
Since L = 0 < 1, the given series converges by root test.
∞ (
∑ )n
n
(b)
n=1
5n + 1
Solution
( )n ( )n 1/n ( )
n √ n n
Here, an = ⇒ n
|an | = = . By root test, we
5n + 1 5n + 1 5n + 1
have
( )
√ n D.S ∞
lim n an = lim = (indeterminate)
n→∞ n→∞ 5n + 1 ∞
[ ]
L’H 1 D.S 1
= lim =
n→∞ 5 5
(1)
1
Since L = < 1, the given series converges by root test.
5

∑ n3
(c)
n=1
3n
Solution

22
3.2 Series ⃝Francis
c Oketch

( )1/n
n3 √ n3 n3/n

Here, an = n ⇒ n
|an | = n = . By root test, we have
3 3 3

√ n3/n 1[ ]
D.S
lim n
an = lim = lim n3/n = ∞0
n→∞ n→∞ 3 3 n→∞

3 3 ln n
Let y = n3/n ⇒ ln y = ln(n) = . Taking limit as n → ∞ on both sides yields
n n
[ ] [ ]
3 ln n D.S ∞
ln lim y = lim = (indeterminate)
n→∞ n→∞ n ∞
[ ]
L’H 3/n D.S 3
= lim = =0
n→∞ 1 ∞
∴ lim y = e0 = 1 ⇒ lim n3/n = 1
n→∞ n→∞

n3/n 1 1 1
Therefore, lim = [1] = . Since L = < 1, the given series converges by root test.
n→∞ 3 3 3 3

(3) Integral test (Maclaurin-Cauchy test)




Let an be a series with positive terms i.e., an > 0 for all n ≥ N . Suppose that f (x) is
n=N
a continuous, positive, and decreasing function on the interval [N, ∞) (for some N ≥ 1) and


f (n) = an . Then, the series an :
n=N

∫∞
i) converges if the improper integral f (x)dx is finite and positive.
N
∫∞
ii) diverges if the improper integral f (x)dx is finite and negative or infinite.
N

→ Note: the integral test is often effective if f (x) is easy to integrate and an involves logarithmic,
exponential, trigonometric, and inverse of trigonometric functions.

Example(s):

∑ 1
1. Use integral test to show that the harmonic series always diverges.
n=1
n
Solution
1 1 1
Here, an = , N = 1 and f (n) = an = ⇒ f (x) = . Clearly, f (x) is positive
n n x
and continuous on the interval [1, ∞). We see that f (x) is decreasing by examining its
1
derivative i.e., f ′ (x) = − 2 < 0 for all x ≥ 1. Thus, by the integral test, we have
x
∫∞ ∫∞ ∫b
1 1
f (x)dx = dx = lim dx
x b→∞ x
N 1 1
( b )

= lim ln x = lim [ln b − ln 1] = lim (ln b)
b→∞ 1 b→∞ b→∞
D.S
= ∞

Since the integral is infinite, it follows that the series an diverges (which confirms that
harmonic series always diverges).

23
3.2 Series ⃝Francis
c Oketch


∑ 1
2. Use integral test to investigate the convergence of the p-series defined by .
n=1
np
Solution
1 1 1
Here, an = , N = 1 and f (n) = an = p ⇒ f (x) = p . Clearly, f (x) is positive
np n x
and continuous on the interval [1, ∞). We see that f (x) is decreasing by examining its
p
derivative i.e., f ′ (x) = − (p+1) < 0 for all x ≥ 1. Thus, by the integral test, we have
x
[ ] 
∫∞ ∫∞ ∫b ∫b  x−p+1 b 
1 1
f (x)dx = dx = lim dx = lim x−p dx = lim
xp b→∞ xp b→∞ b→∞  −p + 1 
N 1 1 1 1
[ ]b  {[
  ]b }
1 1 1
= lim x−(p−1) = lim
1 − p b→∞   1 − p b→∞ xp−1 1
1
[ ] [ ]
1 1 1 1 1
= lim p−1 − p−1 = lim p−1 − 1
1 − p b→∞ b 1 1 − p b→∞ b
[ ]
D.S 1 1
= 0−1 =
1−p p−1

(i) For all p > 1, then the integral is positive and finite. For example, let p = 4 ⇒
1 1
= < ∞. Hence, p-series converges.
p−1 3
(ii) If p ≤ 1, then the integral is negative or infinite. For example, let
1 1 1 1
p = −1 ⇒ = − = −0.5 or let p = 1 ⇒ = = ∞ . Hence, p-series
−1 − 1 2 1−1 0
diverges.
3. Test for convergence of the following series using integral test.

∑ 1
(a)
n=1
n2 +1
∑∞
1/n
(b) √
(ln n) ln2 n − 1
n=3
∑∞
en
(c)
n=1
1 + e2n

∑ 8 tan−1 (n)
(d)
n=1
n2 + 1
∑∞
(e) sechn
n=1
∑∞
ln n
(f)
n=1
n

(4) Limit Comparison test


∑ ∑
Check whether the infinite series an is similar in appearance to another infinite series bn
whose convergence or divergence is known (geometric series, p-series, and harmonic series)
(a ) with
n
an , bn ≥ 0 for all n and apply the limit comparison test which states that if lim = L,
n→∞ bn

where L is finite and positive. Then, it follows that the series an

(i) converges if bn converges.

(ii) diverges if bn diverges.

24
3.2 Series ⃝Francis
c Oketch

Example(s):
Use limit comparison test to investigate the convergence of the following series.

( )
∑ 1
(a)
n=1
2 −5
n

Solution
1 1 ∑
Here, an = n and bn = n . Clearly, bn is a geometric series with common ratio
2 −5 2
1 1 ∑
r = . Since |r| = < 1, the series bn converges. So,
2 2
( ) ( )
(a ) ( 1 2n ) 2n 1
n
lim = lim · = lim = lim
n→∞ bn n→∞ 2n − 5 1 n→∞ 2n − 5 n→∞ 1 − 25n
D.S 1
= 5 =1
1− ∞

Since the limit L = 1 is finite and positive, the series an will have the same convergence

properties as bn . Thus, the given series converges.

∑ 1 1
(b) n
[hint: bn = ]
n=1
n3 3n

∑ n3 1
(c) [hint: bn = ]
n=1
2n4 +1 n
∑∞
1 1
(d) √ [hint: bn = ]
n=1 n3 + 3 n3/2

3.2.4 nth -term test for divergence of an infinite series



Let an be an infinite series and suppose that lim an = L. Then, the series diverges if L ̸= 0.
n→∞

→ Note: if L = 0, then revert to other tests to investigate the convergence/divergence of the given
infinite series.

Example(s):
Use the nth term divergence test to investigate the divergence of the following infinite series.

∑ ( )
2n + 1
(a) ln
n=1
n+1

Solution
( )
2n + 1
Here, an = ln . By the nth term divergence test, we have
n+1
( ) ( ) ( )
2n + 1 2n + 1 D.S ∞
lim an = lim ln = ln lim = ln
n→∞ n→∞ n+1 n→∞ n + 1 ∞
( )
L′ H 2
= ln lim = ln 2
n→∞ 1

Since ln 2 ̸= 0, the given series diverges by the nth -term divergence test.

Exercise:
Use the nth term test for divergence to test for divergence of the following series

∑ 3n
(a)
n=1
2n + 5

25
⃝Francis
c Oketch

∞ (
∑ )n
n
(b) . [ans: L = e−5 is tangible, diverges]
n=1
n+5

Lecture 5

4 Power Series
For purposes of elementary calculus, the most important series of variable terms are those of the form



an (x. − x0 )n ,
n=0

where x0 and the series coefficients an are real numbers. This type of series is called a power series
because its terms proceed in powers of the binomial (x − x0 ). The assumption is that (x − x0 )0 = 1
even if (x − x0 ) = 0.

4.1 Convergence of a power series


The interval in which the power series would converge depends on

(i) The series coefficients an ,

(ii) The point x0 about which the series is expanded, and

(iii) x itself.

Now, to find the radius of convergence of the power series centered at x0 , we consider the following
two cases.

a n+1
n+1 (x − x0 )
CASE 1: By ratio test, the power series will converge if lim < 1. So,
n→∞ an (x − x0 )n

a n
n+1 (x − x0 )(x − x0 )
⇒ lim <1
n→∞ an (x − x0 )n

a
n+1
⇒ lim (x − x0 ) < 1
n→∞ an

a
n+1
⇒ lim x − x0 < 1
n→∞ an

a
n+1
⇒ |x − x0 | lim <1
n→∞ an

1
⇒ |x − x0 | <
a
n+1
lim
n→∞ an

a
n
⇒ |x − x0 | < lim
n→∞ an+1

Thus by ratio test, the radius of convergence of a power series is defined by



a
n
r = lim.
n→∞ an+1

26
4.1 Convergence of a power series ⃝Francis
c Oketch



CASE 2: By nth root test, the power series will converge if lim n an (x − x0 )n < 1. So,
n→∞
1/n

⇒ lim an (x − x0 )n <1
n→∞
⇒ lim |an |1/n |x − x0 | < 1
n→∞
⇒ |x − x0 | lim |an |1/n < 1
n→∞
1
⇒ |x − x0 | <
lim |an |1/n
n→∞

1

⇒ |x − x0 | < lim √
n→∞ n an

Thus by nth root test, the radius of convergence of a power series is defined by

1

r = lim. √
n→∞ n an

The interval of convergence of a power series is given by |x − x0 | < r. This implies that −r <
x − x0 < r. Therefore,
−r + x0 < .x < r + x0

Example(s):

1. Using ratio test, determine the radius of convergence and interval of convergence for the following
power series
∑∞
(−1)n (x − 1)n
n=0
22n

Solution
(−1)n (−1)n+1 (−1)(−1)n
an = , a n+1 = = and x0 = 1. So by ratio test, the radius of
22n 22(n+1) 4(22n )
convergence is

a (−1)n 4(22n )
n
r = lim = lim 2n · = lim | − 4| = 4
n→∞ an+1 n→∞ 2 (−1)(−1)n n→∞

The interval of convergence is given by

−r + x0 < x < r + x0 ⇒ −4 + 1 < x < 4 + 1 ⇒ −3 < x < 5

2. Using nth root test, determine the radius of convergence and interval of convergence for the
following power series
∑∞ ( n + 1 )n
(−1)n (x − 2)n
n=0
5n + 3

Solution
( n + 1 )n
an = (−1)n and x0 = 2. So by nth root test, the radius of convergence is
5n + 3
1/n 1/n
1 −(5 + 3/n)
1 1
r = lim = lim ( n + 1 )n = lim
( n + 1 ) n→∞
= lim =5
n→∞ an n→∞ n→∞ 1 + 1/n
(−1)n (−1)
5n + 3 5n + 3
The interval of convergence is given by

−r + x0 < x < r + x0 ⇒ −5 + 2 < x < 5 + 2 ⇒ −3 < x < 7

27
4.2 Taylor’s and Maclaurin’s series ⃝Francis
c Oketch


∑ 2n
3. Determine the radius and interval of convergence of the power series (4x − 8)n . [ans:
n=1
n
r = 18 , 15
8 <x<
17
8 ]

4. Using either ratio test or nth root test, determine the radius of convergence and interval of
convergence for the following power series.

∑ xn
(a) [ans: r = 3, −3 < x < 3]
n=0
3n(n + 1)

∑ (x + 1)n
(b) [ans: r = 4, −5 < x < 3]
n=1
n4(n−1)
∞ √ √ √
∑ 2n xn 5 5 5
(c) √ [ans: r = ,− <x< ]
n=0
4n+1 10 10 10
5n
∞ √ √ √
∑ 2n xn 3 3 3
(d) √ [ans: r = ,− <x< ]
n=0 (2n + 1)2 3n 2 2 2
∑∞
(x − 4)n
(e) √ [ans: r = 1, 3 < x < 5]
n=0
n+1

∑ (n!)2 (x + 3)n
(f) [ans: r = 4, −7 < x < 1]
n=1
(2n)!
∑∞
(x + 3)n
(g) [ans: r = 5, −8 < x < 2]
n=0
n5n

4.2 Taylor’s and Maclaurin’s series


Consider the function defined by

f (x) = A + B(x − x0 ) + C(x − x0 )2 + D(x − x0 )3 + E(x − x0 )4 + F (x − x0 )5 + · · · (2)

Suppose the function f (x) has a power series representation about x = x0 . Also, assume that f (x)
has derivatives of every order. We need to find the constant coefficients A, B, C, D, E and F , then
substitute in the given function.

(i) Plugging x = x0 in equation (2) yields A = f (x0 ).


(ii) Differentiating equation (2) with respect to x, we obtain

f ′ (x) = B + 2C(x − x0 ) + 3D(x − x0 )2 + 4E(x − x0 )3 + 5F (x − x0 )4 + · · · (3)

Setting x = x0 in equation (3) yields B = f ′ (x0 ).


(iii) Differentiating equation (3) with respect to x, we obtain

f ′′ (x) = 2C + 6D(x − x0 ) + 12E(x − x0 )2 + 20F (x − x0 )3 + · · · (4)


f ′′ (x0 ) f ′′ (x0 )
Setting x = x0 in equation (4) yields C = = .
2 2!
(iv) Differentiating equation (4) with respect to x, we obtain

f ′′′ (x) = 6D + 24E(x − x0 ) + 60F (x − x0 )2 + · · · (5)


f ′′′ (x0 ) f ′′′ (x0 )
Setting x = x0 in equation (5) yields D = = .
6 3!
(v) Differentiating equation (5) with respect to x, we obtain

f (4) (x) = 24E + 120F (x − x0 ) + · · · (6)

f (4) (x0 ) f (4) (x0 )


Setting x = x0 in equation (6) yields E = = .
24 4!

28
4.2 Taylor’s and Maclaurin’s series ⃝Francis
c Oketch

(vi) Differentiating equation (6) with respect to x, we obtain

f (5) (x) = 120F + · · · (7)

f (5) (x0 ) f (5) (x0 )


Setting x = x0 in equation (7) yields F = = .
120 5!
Substituting the values of the constants A, B, C, D, E and F into equation (2) yields

∑∞
f ′′ (x0 ) f ′′′ (x0 ) f (n) (x0 )
f (x) = f (x0 ) + f ′ (x0 )(x − x0 ) + (x − x0 )2 .+ (x − x0 )3 + · · · = (x − x0 )n (8)
2! 3! n=0
n!

Equation (8) is the Taylor’s series expansion of the function f (x) about the point x = x0 . Setting
x0 = 0, we obtain

∑∞
f ′′ (0) 2 . f ′′′ (0) 3 f (n) (0) n
f (x) = f (0) + f ′ (0)x + x + x + ··· = x (9)
2! 3! n=0
n!

Equation (9) is the Maclaurin’s series expansion of the function f (x). So, the Maclaurin’s series
is obtained when a function f (x) is expanded about the point x = 0. It is a special case of Taylor’s
series.

Example(s):

1. Obtain the Maclaurin’s series expansion of the following functions.

(a) f (x) = e−2x .

Solution
The Maclaurin’s series expansion for a function f (x) is given by

f ′′ (0) 2 f ′′′ (0) 3 f (4) (0) 4


f (x) = f (0) + f ′ (0)x + x + x + x + ···
2! 3! 4!
Thus we expand f (x) about the point x = 0.

f (x) = e−2x f (0) = 1


′ −2x
f (x) = −2e f ′ (0) = −2
f ′′ (x) = 4e−2x f ′′ (0) = 4
f ′′′ (x) = −8e−2x f ′′′ (0) = −8
f (4) (x) = 16e−2x f (4) (0) = 16

Hence,
4 2 8 16
f (x) = 1 − 2x + x − x3 + x4 + · · ·
2! 3! 4!
∑∞
4 2 (−2x)n
∴ e−2x = 1 − 2x + 2x2 − x3 + x4 + · · · =
3 3 n=0
n!

∑∞
x3 x5 x7 (−1)n x2n+1
(b) f (x) = sin x [ans: sin x = x − + − + ··· = ]
3! 5! 7! n=0
(2n + 1)!
∑∞
x2 x3 x4 (−1)n xn+1
(c) f (x) = ln(1 + x) [ans: ln(1 + x) = x − + − + ··· =
2 3 4 n=0
n+1
( )
n(n − 1) 2 ∑n
n r
(d) f (x) = (1 + x)n [ans: (1 + x)n = 1 + nx + x + · · · + xn = x ]
2! r=0
r

29
4.2 Taylor’s and Maclaurin’s series ⃝Francis
c Oketch

x3 x5
(e) f (x) = sinh x. [ans: sinh x = x + + + ···]
3! 5!

2. Use Maclaurin’s series to expand cos x in ascending powers of x upto the term containing x6 .
Hence, use your expansion to evaluate cos(66o ) correct to five decimal places.

Solution
The Maclaurin’s series expansion for a function f (x) is given by

f ′′ (0) 2 f ′′′ (0) 3 f (4) (0) 4 f (5) (0) 5 f (6) (0) 6


f (x) = f (0) + f ′ (0)x + x + x + x + x + x + ···
2! 3! 4! 5! 6!
Thus we expand f (x) about the point x = 0.

f (x) = cos x f (0) = 1



f (x) = − sin x f ′ (0) = 0
f ′′ (x) = − cos x f ′′ (0) = −1
f ′′′ (x) = sin x f ′′′ (0) = 0
f (4) (x) = cos x f (4) (0) = 1
f (5) (x) = − sin x f (5) (0) = 0
f (6) (x) = − cos x f (6) (0) = −1

Hence,
x2 x4 x6
cos x = 1 − + − + ···
2! 4! 6!
Therefore,
(11π/30)2 (11π/30)4 (11π/30)6
cos(66o ) = cos(11π/30) ≈ 1 − + −
2! 4! 6!
≈ 0.40666 (5 d.p.)

3. Obtain the Taylor’s series expansion of the following functions about the indicated points.

(a) f (x) = ex/2 about x0 = 2.

Solution
The Taylor’s series expansion for a function f (x) about the point x = x0 is given by
f ′′ (x0 ) f ′′′ (x0 )
f (x) = f (x0 ) + f ′ (x0 )(x − x0 ) + (x − x0 )2 + (x − x0 )3 + · · ·
2! 3!
Setting x0 = 2 yields
f ′′ (2) f ′′′ (2)
f (x) = f (2) + f ′ (2)(x − 2) + (x − 2)2 + (x − 2)3 + · · ·
2! 3!
Now,

f (x) = ex/2 f (2) = e


1 1
f ′ (x) = ex/2 f ′ (2) = e
2 2
′′ 1 x/2 ′′ 1
f (x) = e f (2) = e
4 4
1 1
f ′′′ (x) = ex/2 f ′′′ (2) = e
8 8
(4) 1 x/2 (4) 1
f (x) = e f (2) = e
16 16

30
4.2 Taylor’s and Maclaurin’s series ⃝Francis
c Oketch

Hence,
e e e e
f (x) = e + (x − 2) + (x − 2)2 + (x − 2)3 + (x − 2)4 + · · ·
2 4(2!) 8(3!) 16(4!)
[ 1 1 1 1 ]
∴ ex/2 = e 1 + (x − 2) + (x − 2)2 + (x − 2)3 + (x − 2)4 + · · ·
2 8 48 384
(b) f (x) = ln x about the point x = 2.
π
(c) f (x) = sin−1 x about the point x = .
4
1
(d) f (x) = e−8x about the point x = .
2

Exercise:
1. Expand e−x using Maclaurin’s series upto the 4th term. Hence, use your expansion to evaluate
1
correct to 4dp. [ans: = 0.3679]
e
2. Use your knowledge of Maclaurin’s series to approximate the following correct to 4dp. [hint:
convert the given degrees into radian using 180o = π c ]

(a) e−2 [ans: = 0.1353]


(b) sin 62o [ans: = 0.8829]
(c) cos 36o
(d) tan 31o
(e) ln(0.97)

3. Using series expansion, evaluate the following


( )
x 2 ex
(a) lim [ans: = −2]
x→0 cos x − 1
( )
sin x − x 1
(b) lim [ans: = − ]
x→0 x3 6
( )
cos x − 1 + 12 x2 1
(c) lim [ans: = ]
x→0 x4 24
1
4. (a) Obtain the Maclaurin’s series expansion of (i) cos x and (ii) . Hence, show that
1+x
cos x x2 x3 13 4
=1−x+ − + x + ···
1+x 2 2 24

x x x2 x4
(b) Show that = − + − ···
ex + 1 2 4 48
(c) Obtain the Maclaurin’s series expansion
√ of (i) cos θ, (ii) sin θ, and (iii) eθ . Hence, prove the
following formulae (where i = −1):
 eiθ = cos θ + i sin θ and e−iθ = cos θ − i sin θ.
eiθ + e−iθ eiθ − e−iθ
 cos θ = and sin θ = .
2 2i
(d) Obtain the Maclaurin’s series expansion of (i) cosh θ, (ii) sinh θ, and (iii) eθ . Hence, prove
the following formulae:
eθ + e−θ eθ − e−θ
 cosh θ = and sinh θ = .
2 2
CAT 1

Lecture 6

31
⃝Francis
c Oketch

5 Differentiation
5.1 Partial derivatives
Let z be a function of two independent variables x and y i.e., z = z(x, y). Since x and y are independent,
it is possible to:

i) Allow x to vary while y is fixed

ii) Allow y to vary while x is fixed

iii) Allow x and y to vary simultaneously

From the above, we may define the partial derivatives of the function z by
[ ] [ ]
∂z z(x + ∆x, y) − z(x, y) ∂z z(x, y + ∆y) − z(x, y)
= lim and = lim
∂x ∆x→0 ∆x ∂y ∆y→0 ∆y

∂z
if these limits exist. To compute the partial derivative of z with respect to x (i.e., ), we hold
∂x
(keep) y constant. Also, to compute the partial derivative of z with respect to y we keep x constant.

∂z
→ Note: can also be written in subscript notation as zx . Similarly,
∂x
∂z ∂2z ∂2z ∂2z ∂2z
≡ zy , ≡ zxx , ≡ zyy , ≡ zxy , ≡ zyx
∂y ∂x2 ∂y 2 ∂x∂y ∂y∂x

Example(s):

1. If z = 2x2 − 3xy + 4y 2 , find zx , zy , zxx , zyy , zxy , and zyx .

Solution
∂z ∂z
zx = = 4x − 3y, zy = = −3x + 8y
∂x ∂y
∂2z ∂ ∂2z ∂
zxx = 2
= (4x − 3y) = 4 zyy = 2
= (−3x + 8y) = 8
∂x ∂x ∂y ∂y
∂2z ∂ ∂2z ∂
zxy = = (−3x + 8y) = −3 zyx = = (4x − 3y) = −3
∂x∂y ∂x ∂y∂x ∂y

∂2z ∂2z
→ Note: in differential calculus, zxy = zyx i.e., = .
∂x∂y ∂y∂x
2. (a) Find zx and zy given that xy + yz 2 + zx = 1.

Solution
To find zx , differentiate the given equation implicitly with respect to x treating y as a
constant. Thus,

(y + z)
y + 2yzzx + z + xzx = 0 ⇒ zx = −
2yz + x
Similarly, differentiating the given equation implicitly with respect to y treating x as a
constant yields

(x + z 2 )
x + 2yzzy + z 2 + xzy = 0 ⇒ zy = −
2yz + x

32
5.1 Partial derivatives ⃝Francis
c Oketch

2 +y 2
(b) Given that z = ex , find zx , zy , zxx and zyy .

Solution
∂z ∂ ( 2
2 +y 2
) 2 2
= ex ·
x + y 2 = 2xex +y
∂x ∂x
∂z 2 2 ∂ ( 2 ) 2 2
= ex +y · x + y 2 = 2yex +y
∂y ∂y
[ ]
∂2z ∂ ∂z ∂ [ 2 2
] { 2 2 2 2
} 2 2
( )
= = 2xex +y = 2 2x2 ex +y + ex +y = 2ex +y 2x2 + 1
∂x2 ∂x ∂x ∂x
[ ]
∂2z ∂ ∂z ∂ [ x2 +y2 ] { 2 2 2 2
} 2 2
( )
= = 2ye = 2 2y 2 ex +y + ex +y = 2ex +y 2y 2 + 1
∂y 2 ∂y ∂y ∂y

∂2z ∂2z
(c) Given z = ln(x2 + y 2 ), show that + = 0.
∂x2 ∂y 2
Solution
∂z 1 ∂ ( 2 ) 2x
= · x + y2 = 2
∂x x + y2
2 ∂x x + y2
∂z 1 ∂ ( ) 2y
= · x2 + y 2 = 2
∂y x + y2
2 ∂y x + y2
[ ] [ ] ∂ ∂
∂2z ∂ ∂z ∂ 2x (x2 + y 2 ) (2x) − 2x (x2 + y 2 ) −2x2 + 2y 2
= = = ∂x ∂x =
∂x2 ∂x ∂x ∂x x2 + y 2 (x2 + y 2 )2 (x2 + y 2 )2
∂ ∂
[ ] [ ] (x2 + y 2 ) (2y) − 2y (x2 + y 2 )
∂2z ∂ ∂z ∂ 2y ∂y ∂x 2x2 − 2y 2
= = = =
∂y 2 ∂y ∂y ∂y x2 + y 2 (x2 + y 2 )2 (x2 + y 2 )2

∂2z ∂2z −2x2 + 2y 2 2x2 − 2y 2


Therefore, + = + =0
∂x2 ∂y 2 (x2 + y 2 )2 (x2 + y 2 )2

Exercise:
( 25xy )
1. Given z = tan−1 , find zx and zy .
x2 + y 2

Solution
( 25xy ) 25xy
z = tan−1 ⇒ tan z = (10)
x2 + y2 x2 + y 2

Differentiating equation (10) partially with respect to x [keeping y constant], we get

(x2 + y 2 )(1) − x(2x) 25y(y 2 − x2 )


(sec2 z)zx = 25y =
(x2 + y 2 )2 (x2 + y 2 )2
( )2
25xy (x2 + y 2 )2 + 625x2 y 2
But sec2 z = 1 + tan2 z. So, 1 + = . Hence,
x2 + y 2 (x2 + y 2 )2
(x2 + y 2 )2 + 625x2 y 2 25y(y 2 − x2 )
z x = . Therefore,
(x2 + y 2 )2 (x2 + y 2 )2

25y(y 2 − x2 )
zx =
(x2 + y 2 )2 + 625x2 y 2

Similarly, differentiating equation (10) partially with respect to y [keeping x constant], we get

(x2 + y 2 )(1) − y(2y) 25x(x2 − y 2 )


(sec2 z)zy = 25x =
(x2 + y 2 )2 (x2 + y 2 )2

33
5.2 Total differentials and total derivatives ⃝Francis
c Oketch

(x2 + y 2 )2 + 625x2 y 2 (x2 + y 2 )2 + 625x2 y 2 25x(x2 − y 2 )


But sec2 z = . Hence, zy = . Therefore,
(x2 + y 2 )2 (x2 + y 2 )2 (x2 + y 2 )2
25x(x2 − y 2 )
zy =
(x2 + y 2 )2 + 625x2 y 2
2. Find zx , zy , zxx and zyy given that
(a) z = sin(2x + 3y)
(b) x2 + y 2 + z 2 = 25
(c) x2 (2y + 3z) + y 2 (3x − 4z) = 10
10yz − 8(x3 + y 3 z) + 12x2 yz
(d)* x3 (2x − 4yz) + y 3 (3y + 8xz) = 10xyz [ans: zx = ]
8xy 3 − 4x3 y − 10xy
(√ ) ∂2f ∂2f ∂2f
3. Given that f = ln x2 + y 2 + z 2 , find + + .
∂x2 ∂y 2 ∂z 2
4. (a) If z = ex/y sin(x/y) + ey/x cos(y/x), show that xzx + yzy = 0.
∂2R ∂2R ∂2R
(b) If R = ln(x2 + y 2 + z 2 ), show that x =y =z .
∂y∂z ∂z∂x ∂x∂y
∂2z ∂2z
(c) If z = f (x + 2y) + g(x − 2y), show that 4 2 = 2 .
∂x ∂y
( )
2xy ∂z ∂z
(d) If z = tan−1 , show that x +y = 0.
x2 − y 2 ∂x ∂y
(√ )
(e) If z = ln x2 − y 2 , show that xzx + yzy = 1.

5.2 Total differentials and total derivatives


Consider the function z = z(x, y) of two independent variables x and y and define dx = ∆x and
dy = ∆y. When x varies while y is fixed, then z is a function of x only and the partial differential of z
∂z
with respect to x is defined as dx. Similarly, the partial differential of z with respect to y is defined
∂x
∂z
as dy. Hence, the total differential of z (i.e., dz) is defined as the sum of the partial differentials
∂y
i.e.,
∂z . ∂z
dz = dx + dy
∂x ∂y

Example(s):
1. Find the total differential of z given that
(a) z = x3 y + x2 y 2 + xy 3

Solution
∂z ∂z
Since z = z(x, y), ⇒ dz =dx + dy.
∂x ∂y
∂z ∂z
Now, = 3x2 y + 2xy 2 + y 3 and = x3 + 2x2 y + 3xy 2 . Substituting yields
∂x ∂y
dz = (3x2 y + 2xy 2 + y 3 )dx + (x3 + 2x2 y + 3xy 2 )dy
(b) z = x sin y − y sin x

Solution
∂z ∂z
Since z = z(x, y), ⇒ dz =
dx + dy.
∂x ∂y
∂z ∂z
Now, = sin y − y cos x and = x cos y − sin x. Substituting yields
∂x ∂y
dz = (sin y − y cos x)dx + (x cos y − sin x)dy

34
5.2 Total differentials and total derivatives ⃝Francis
c Oketch

y xdy − ydx
2. i) Find dθ if θ = tan−1 (y/x). [hint: tan θ = , ans: dθ = ]
x x2 + y 2
ii) Find dz if z = sin−1 (y/x). [ans: ]
2 −y 2
3. Find dz if z = ex . [hint: ln z = x2 − y 2 , ans: dz = 2z(xdx − ydy)]

Suppose z = z(x, y) is a continuous function of two independent variables x and y with continuous
∂z ∂z
partial derivatives i.e., and . If x and y are differentiable functions of the independent variable t
∂x ∂y
dz
i.e., x = x(t) and y = y(t), then z is a function of t by extension and is called the total derivative
dt
of z with respect to t. Therefore,
dz ∂z dx ∂z dy
= +
dt ∂x dt ∂y dt
In general, if z = z(x1 , x2 , x3 , · · · , xn ) where the independent variables are themselves functions of t
i.e x1 = x1 (t), x2 = x2 (t), x3 = x3 (t), · · · , xn = xn (t), then the total derivative of z with respect
to t is given by
dz ∂z dx1 ∂z dx2 . ∂z dx3 ∂z dxn
= + + + ··· +
dt ∂x1 dt ∂x2 dt ∂x3 dt ∂xn dt

Example(s):
dz
1. Given that z = x2 + 3xy + 5y 2 , x = sin t and y = cos t. Find .
dt

Solution
dz ∂z dx ∂z dy
Since z = z(x, y) and the independent variables are functions of t, then = + .
dt ∂x dt ∂y dt
∂z ∂z dx dy
Now, = 2x + 3y, = 3x + 10y, = cos t, and = − sin t. Substituting yields
∂x ∂y dt dt
dz
= (2x + 3y) cos t − (3x + 10y) sin t = (2 sin t + 3 cos t) cos t − (3 sin t + 10 cos t) sin t
dt
= 2 sin t cos t + 3 cos2 t − 3 sin2 t − 10 sin t cos t = −8 sin t cos t + 3(cos2 t − sin2 t)
= −4 sin 2t + 3 cos 2t

dz ( )
2. Find given z = ln x2 + y 2 , x = e−t and y = et .
dt

Solution
dz ∂z dx ∂z dy
Since z = z(x, y) and the independent variables are functions of t, then = + .
dt ∂x dt ∂y dt
dz 1 1 ∂u ∂u
Let z = ln (u) where u = x2 + y 2 . Thus, = = 2 2
, = 2x and = 2y.
du u x + y ∂x ∂x
∂z dz ∂u 2x ∂z dz ∂u 2y
Now, chain rule yields = = 2 2
and = = 2 .
∂x du ∂x x +y ∂y du ∂y x + y2
dx dy
Also, = −e−t and = et . Substituting yields
dt dt
dz ( )(
2x
) ( ) (
2y
)
−2xe−t + 2yet
= −e−t + e t
=
dt x2 + y 2 x2 + y 2 x2 + y 2
−t −t
( 2t −2t
)
−2e e + 2e e t t 2 e −e
= −2t
= = 2 tanh(2t)
e +e 2t e2t + e−2t

dz dz ( )
3. Find given that z = x2 + 2xy + 4y 2 and y = e4x . [ans: = 2x + 2y + (2x + 8y) 4e4x ]
dx dx
du 1 du
4. Find given that u = xy + yz + zx, y = and z = x2 . [ans: = 3x2 + 1]
dx x dx

35
5.2 Total differentials and total derivatives ⃝Francis
c Oketch

However, if x and y are continuous functions such that x = x(r, s) and y = y(r, s) of the independent
variables r and s, then z is a function of r and s by extension. Therefore, the partial derivative of z
with respect to r is defined by
∂z ∂z ∂x ∂z ∂y
= +
∂r ∂x ∂r ∂y ∂r
Also, the partial derivative of z with respect to s is defined by
∂z ∂z ∂x ∂z ∂y
= +
∂s ∂x ∂s ∂y ∂s

In general, if z = z(x1 , x2 , x3 , · · · , xn ) where the independent variables are themselves functions of r


and s i.e x1 = x1 (r, s), x2 = x2 (r, s), x3 = x3 (r, s), · · · , xn = xn (r, s), then the partial derivative
of z with respect to r is given by

∂z ∂z ∂x1 ∂z ∂x2 . ∂z ∂x3 ∂z ∂xn


= + + + ··· +
∂r ∂x1 ∂r ∂x2 ∂r ∂x3 ∂r ∂xn ∂r

and the partial derivative of z with respect to s is given by

∂z ∂z ∂x1 ∂z ∂x2 . ∂z ∂x3 ∂z ∂xn


= + + + ··· +
∂s ∂x1 ∂s ∂x2 ∂s ∂x3 ∂s ∂xn ∂s

Example(s):
∂z ∂z
1. Find and given that z = x2 + xy + y 2 , x = 2r + s and y = r − 2s.
∂r ∂s

Solution
Since z = z(x, y) and the independent variables are functions of r and s, then

∂z ∂z ∂x ∂z ∂y ∂z ∂z ∂x ∂z ∂y
= + and = + .
∂r ∂x ∂r ∂y ∂r ∂s ∂x ∂s ∂y ∂s

∂z ∂z ∂x ∂x ∂y ∂y
Now, = 2x + y, = x + 2y, = 2, = 1, = 1, and = −2. Substituting yields
∂x ∂y ∂r ∂s ∂r ∂s
∂z
= (2x + y)(2) + (x + 2y)(1) = 5x + 4y = 5(2r + s) + 4(r − 2s) = 14r − 3s
∂r
∂z
= (2x + y)(1) + (x + 2y)(−2) = −3y = −3(r − 2s) = −3r + 6s
∂s

∂z ∂z
2. Find and if z = sin(5y + 4x), x = r + s, and y = s − r.
∂r ∂s
∂u ∂u ∂u
3. Find , , and given that u = x2 + 2y 2 + 2z 2 , x = ρ sin β cos θ, y = ρ sin β sin θ and
∂ρ ∂β ∂θ
∂u 2u ∂u ∂u
z = ρ cos β. [ans: = , = 2x2 cot β + 4y 2 cot β − 4z 2 tan β, = 2xy]
∂ρ ρ ∂β ∂θ
( )2 ( )2 ( )2 ( )2
∂u ∂u ∂u 1 ∂u
4. If u = u(x, y) and x = r cos θ, y = r sin θ, show that + = + 2
∂x ∂y ∂r r ∂θ

5. Suppose f is a twice differentiable real function. Deduce that if z = f (r) where r = x2 + y 2
∂2z ∂2z 1
then z satisfies the equation 2
+ 2
= f ′ (r) + f ′′ (r).
∂x ∂y r

36
5.2 Total differentials and total derivatives ⃝Francis
c Oketch

5.2.1 Applications of total derivatives

Example(s):
1. Approximate the change in hypotenuse of a right angled triangle of base 60mm and height 80mm
if the base is increased by 2.5mm and the height is decreased by 1.25mm.

Solution
Let x, y, z be the base, height, and hypotenuse of the right angled
triangle, respectively. Then by Pythagoras theorem, we have
y z √
z= x2 + y 2
.
x Given: x = 60mm, y = 80mm, dx = 2.5mm, and dy = −1.25mm.

Therefore, z = (60)2 + (80)2 = 100mm.

∂z ∂z ∂z x
Since z = z(x, y), the change in hypotenuse is given by dz = dx + dy. Now, = and
∂x ∂y ∂x z
∂z y
= . So,
∂y z
∂z ∂z x y 60 80
dz = dx + dy = dx + dy = (2.5) + (−1.25)
∂x ∂y z z 100 100
= 0.5

Therefore, the hypotenuse increases by 0.5mm.


V2
2. Power (P ) consumed in an electrical resistor is given by P = watts. If V = 200 volts and
R
R = 8 ohms. By how much does the power change if V is decreased by 5 volts and R is decreased
by 0.2 ohms.

Solution
We are given that V = 200, R = 8, dV = −5, and dR = −0.2. Since P = P (V, R), the change in
power is given by
∂P ∂P
dP = dV + dR.
∂V ∂R
∂P 2V ∂P V2
Now, = and = − 2 . So,
∂V R ∂R R
∂P ∂P 2V V2 2(200) (200)2
dP = dV + dR = dV − 2 dR = (−5) − (−0.2)
∂V ∂R R R 8 (8)2
= −125

Therefore, the power P decreases by 125 watts.

Exercise:
1. The dimensions of a rectangular block of wood were found to be 100mm, 120mm, and 200mm
with possible error of 5mm in each of the measurements. Approximate the greatest error in
the surface area of the block and the percentage error in the area caused by the errors in the
individual measurements.

Solution
Let l, w, h be the length, width, and height of the block, respectively. Then the surface area is
given by S = 2(lw + lh + wh). We are given that l = 120mm, w = 100mm, h = 200mm, and
dl = dw = dh = ±5mm.

37
⃝Francis
c Oketch

(i) Since S = S(l, w, h), the change in surface area is given by

∂S ∂S ∂S
dS = dl + dw + dh.
∂l ∂w ∂h
∂S ∂S ∂S
Now, = 2(w + h), = 2(l + h), and = 2(l + w). So,
∂l ∂w ∂h
∂S ∂S ∂S
dS = dl + dw + dh
∂l ∂w ∂h
= 2(w + h)dl + 2(l + h)dw + 2(l + w)dh
= 2(100 + 200)(±5) + 2(120 + 200)(±5) + 2(120 + 100)(±5)
= ±8400

Therefore, the greatest error in the surface area of the block is ±8400mm2 .
( )
(ii) Original surface area is S = 2 120 × 100 + 120 × 200 + 100 × 200 = 112000mm2 . So,
percentage error is given by
dS ±8400
% error = × 100% = × 100% = ±7.5%
S 112000
2. Two sides of a triangle are measured 150m and 200m and the included angle as 60o . If the
possible errors are 0.2m in measuring the sides and 0.1o in the angle. What is the greatest
1
possible error in the computed area. [hint: A = ab sin θ, ans: = ±780.310889m2 ]
2
3. The voltage V in an electrical resistor is slowly decreasing as the battery wears out. The
resistance R is slowly increasing as the resistor heats up. Given that V = IR, determine how
dV
the current I is changing at the moment when R = 400Ω, I = 0.08A, = −0.01V/s, and
dt
dR dI
= 0.03Ω/s. [ans: = −3.1 × 10−5 A/s]
dt dt
v{ }
u
1 u
t 1 − R
2
4. The frequency of oscillation of LRC series is given by f = . If L is increased
2π LC 4L2
by 1% and C is decreased by 1%, show that the percentage increase in f is approximately
R2 C
.
AL − R2 C
5. The total surface
√ area S of a right cone of radius r and perpendicular height h is given by
S = πr2 + πr r2 + h2 . If r and h are each increasing at the rate of 0.25cm/s, find the rate at
which S is increasing at the instant when r = 3cm and h = 4cm.

Lecture 7

6 Integration
6.1 Improper integrals
∫b
The definite integral f (x)dx is called an improper integral if:
a

i) at least one of the limits of integration is infinite i.e., a = −∞ and/or b = ∞.

ii) the integrand f (x) has at least one point of discontinuity in the interval [a, b].

38
6.1 Improper integrals ⃝Francis
c Oketch

6.1.1 Improper integrals with infinite limits of integration


Consider the first kind of improper integrals in which at least one of the integration limits is infinite.
Because infinity is not a real number, we can’t just integrate as normal and then“plug in” the infinity
to get an answer. Instead, we think of this as an area problem. Thus, we use the following method of
evaluation (pincer method).
∫∞ ∫b
1. If f (x) is continuous on the interval [a, ∞), then f (x)dx = lim f (x)dx.
b→∞
a a

∫b ∫b
2. If f (x) is continuous on the interval (−∞, b], then f (x)dx = lim f (x)dx.
a→−∞
−∞ a

3. If f (x) is continuous on the interval (−∞, ∞), then for any real number c we have

∫∞ ∫c ∫∞ ∫c ∫b
f (x)dx = f (x)dx + f (x)dx = lim f (x)dx + lim f (x)dx
a→−∞ b→∞
−∞ −∞ c a c

→ Note: if the associated limit exists (i.e., the value is finite), then the improper integral is said to
converge to that limiting value. If the associated limit doesn’t exist (i.e., the value is ±∞), then the
integral is said to diverge or to fail to exist.

Example(s):
∫0
1. Evaluate e2x dx.
−∞

Solution
∫0 ∫0 [1 ]0 [ ]
1
2x
e dx = lim e2x dx = lim e2x = lim 1 − e2a
a→−∞ a→−∞ 2 a 2 a→−∞
−∞ a
1[ ] 1[ 1 ] 1[ ] 1
1 − e−2∞ = 1 − 2∞ = 1 − 0 =
D.S
=
2 2 e 2 2
Since the associated limit exists, therefore, the given improper integral converges.
∫∞
2. Evaluate sin xdx.
0

Solution
∫∞ ∫b
sin xdx = lim sin xdx = lim [− cos x]b0 = lim [1 − cos b]
b→∞ b→∞ b→∞
0 0

Since cos b doesn’t approach a limit as b → ∞, therefore, the given improper integral diverges.
∫∞
3. Evaluate (1 − x)e−x dx.
1

Solution
Using integration by parts [order of choosing u is LIATEC],∫ we put u = (1

− x) and dv = e−x dx.
It implies that du = −1dx and v = −e−x . Substituting in udv = uv − vdu, we get
∫ ∫
−x −x
(1 − x)e dx = −e (1 − x) − e−x dx = −e−x (1 − x) + e−x + C = xe−x + C

39
6.1 Improper integrals ⃝Francis
c Oketch

Therefore,
∫∞ [ ] [ ]
−x [ ]b b 1
(1 − x)e dx = lim xe−x 1 = lim be −b
−e −1
= lim b

b→∞ b→∞ b→∞ e e
1
[ ]
D.S ∞ 1 ∞
= ∞
− = (indeterminate)
e e ∞
[ ] [ ]
L’H 1 1 D.S 1 1 1 1
= lim − = − =0− =−
b→∞ eb e e∞ e e e
Since the associated limit exists, therefore, the given improper integral converges.
∫∞
ex
4. Evaluate dx.
1 + e2x
−∞

Solution
The integrand is continuous on (−∞, ∞). To evaluate this integral, we break it into two parts,
choosing c = 0 as a convenient value yields

∫∞ ∫0 ∫∞ ∫0 ∫b
ex ex ex ex ex
dx = dx + dx = lim dx + lim dx
1 + e2x 1 + e2x 1 + e2x a→−∞ 1 + e2x b→∞ 1 + e2x
−∞ −∞ 0 a 0
[ ]0 [ ]b
= lim tan−1 (ex ) + lim tan−1 (ex )
a→−∞ a b→∞ 0
[ ] [ ]
−1 −1
= lim tan (1) − tan (e ) + lim tan−1 (eb ) − tan−1 (1)
a
a→−∞ b→∞
[ ] [ ]
−1 −1
(eb ) = − tan−1 (e−∞ ) + tan−1 (e∞ )
D.S
= − lim tan a
(e ) + lim tan
a→−∞ b→∞
π π
= − tan−1 (0) + tan−1 (∞) = −0 + =
2 2
Since the associated limit exists, therefore, the given improper integral converges.

Exercise:
∫∞
1 1
1. Evaluate dx. [ans: ln(2)]
x(x2 + 1) 2
1

6.1.2 Improper integrals with discontinuous integrands


Consider the second kind of improper integrals in which the integrand f (x) is discontinuous at (or
between) the limits of integration. Then, pincer method yields

1. If f (x) is continuous on the interval [a, b) and is discontinuous at b, then

∫b ∫c
f (x)dx = lim f (x)dx
c→b−
a a

2. If f (x) is continuous on the interval (a, b] and is discontinuous at a, then

∫b ∫b
f (x)dx = lim f (x)dx
c→a+
a c

3. If f (x) is continuous on the interval [a, b], except for some c in (a, b) at which f (x) is
∫b ∫c ∫b ∫p ∫b
discontinuous, then f (x)dx = f (x)dx + f (x)dx = lim f (x)dx + lim f (x)dx.
p→c− p→c+
a a c a p

40
6.1 Improper integrals ⃝Francis
c Oketch

Example(s):
∫2
1
1. Evaluate dx.
x3
0

Solution
1
Clearly, the integrand f (x) = is discontinuous at the point x = 0, which is in the interval
x3
[0, 2]. Therefore,

∫2 ∫2 [ ]2 [ ]
1 1 1 1 1
dx = lim dx = lim = lim − + 2 = ∞
x3 b→0+ x3 b→0 + 2x 2
b b→0 + 8 2b
0 b

Hence, the given improper integral diverges.


∫4
x
2. Evaluate dx.
x2 −9
−1

Solution
x
Clearly, the integrand f (x) = is discontinuous at the point x = 3, which is in the interval
x2 − 9
[−1, 4]. To evaluate this integral, we split it at the point of discontinuity. Therefore,

∫4 ∫3 ∫4
x x x
dx = dx + dx
x −9
2 x −9
2 x2 −9
−1 −1 3
∫a ∫4
x x
= lim dx + lim dx
a→3− x −9
2 b→3+ x2 −9
−1 b
[ a ] 4[ ]
1 1
= lim ln |x2 − 9| + lim ln |x2 − 9|
a→3− 2 −1 b→3+ 2 b
[ ] [ ]
1 1 1 1
= lim ln |a − 9| − ln | − 8| + lim
2
ln |7| − ln |b − 9|
2
a→3− 2 2 b→3+ 2 2
= −∞ (+∞)

Since at least one of the integrals on the right diverges, therefore, the original improper integral
also diverges.
∫∞
1
3. Evaluate √ dx.
x (x + 1)
0

Solution
Here, the integrand is discontinuous at x = 0 and one of the limits of integration is infinite (this
is a doubly improper integral). To evaluate this integral, we split it at a convenient point (say

41
6.1 Improper integrals ⃝Francis
c Oketch

x = 1). Therefore,

∫∞ ∫1 ∫∞
1 1 1
√ dx = √ dx + √ dx
x (x + 1) x (x + 1) x (x + 1)
0 0 1
∫1 ∫c
1 1 √
= lim √ dx + lim √ dx (put x = tan θ)
b→0+ x (x + 1) c→∞ x (x + 1)
b 1
[ √ ]1 [ √ ]c
= lim 2 tan−1 ( x) + lim 2 tan−1 ( x)
b→0+ b c→∞ 1
[ √ ] [ √ ]
= lim 2 tan−1 (1) − 2 tan−1 ( b) + lim 2 tan−1 ( c) − 2 tan−1 (1)
b→0+ c→∞
[ √ ] [ √ ]
= −2 lim tan−1 ( b) + 2 lim tan−1 ( c)
b→0+ c→∞
[ ] [ ] [ ]
D.S −1 −1 π
= −2 tan (0) + 2 tan (∞) = −2 [0] + 2 =π
2
Since the associated limit exists, therefore, the given improper integral converges.
∫1
4. Evaluate ln xdx.
0

Solution
Here, the integrand has infinite discontinuity at x = 0. Using integration by parts, let u = ln x
1 ∫ ∫
and dv = dx. Thus, du = dx and v = x. Substituting in udv = uv − vdu, we get
x
∫ ∫
1
ln xdx = x ln x − x· dx = x ln x − x + C
x
Therefore,
∫1 ( )
ln b
ln xdx = lim [x ln x − x]1b = lim [0 − 1 − b ln b + b] = −1 − lim + lim (b)
b→0+ b→0+ b→0+ 1/b b→0+
0
( )
1/b
= −1 − lim + lim (b) = −1 − 0 + 0 = −1
b→0+ −1/b2 b→0+

Since the associated limit exists, therefore, the given improper integral converges.

Exercise:

1. Evaluate the following improper integrals.


∫∞
(a) x3 e−x dx [ans: = ∞]
−∞
∫∞ √
e− x 2
(b) √ dx [ans: = ]
x e
1
∫0
(c) xex dx [ans: = −1]
−∞
∫∞
1
(d) dx
x ln2 x
2
∫∞
1
(e) dx [hint: complete the square in denominator then use partial fractions]
2x2 + 3x + 1
0

42
6.2 Reduction formulae ⃝Francis
c Oketch

∫∞
1
(f) dx [hint: complete the square, use trigonometric subst.]
x2 + 6x + 10
−∞
∫0
xe−x dx
2
(g) [hint: let t = x2 ]
−∞
∫1
(h) x ln xdx. [ans: −1/4]
0

2. Test for convergence of the following improper integral


∫∞
3x2 e−x dx.
3
(a) [ans: I = 1e , converges]
1
∫∞
x+3
(b) dx. [ans: ]
(x − 1)(x2 + 1)
2

6.2 Reduction formulae


On applications of methods of integration by parts, we can derive the reduction formula for a given
integral. By means of the reduction formula, the given integral is expressed as the sum of the terms,
one without the integral sign and the other an integral of the same form as the original integral but
easier to integrate.

Example(s):

1. Derive the reduction formula for:



(a) In = xn e−x dx hence compute I3 .

Solution
Let u = xn and dv = e−x dx ⇒ du = nxn−1 dx and v = −e−x .
∫ ∫
n −x
x e dx = uv − vdu

= −xn e−x + n xn−1 e−x dx
∴ In = −xn e−x + nIn−1

Now,

I3 = −x3 e−x + 3I2


I2 = −x2 e−x + 2I1
I1 = −xe−x + I0

But I0 = e−x dx = −e−x + C1 . Therefore, back substitution yields:

∴ I3 = −x3 e−x − 3x2 e−x − 6xe−x − 6e−x + C = −(x3 + 3x2 + 6x + 6)e−x



(b) In = sinn xdx hence compute I4 .

Solution

43
6.2 Reduction formulae ⃝Francis
c Oketch

∫ ∫
In = sinn xdx = sinn−1 x sin xdx. Let u = sinn−1 x and dv = sin xdx ⇒ du =
(n − 1) sinn−2 x cos xdx and v = − cos x.
∫ ∫
sin xdx = uv −
n
vdu

= − sinn−1 x cos x + (n − 1) sinn−2 x cos2 xdx

= − sin n−1
x cos x + (n − 1) sinn−2 x[1 − sin2 x]dx
∫ ∫
= − sinn−1 x cos x + (n − 1) sinn−2 xdx − (n − 1) sinn xdx

⇒ In = − sinn−1 x cos x + (n − 1)In−2 − (n − 1)In


sinn−1 x cos x (n − 1)
∴ In = − + In−2
n n
Now,

sin3 x cos x 3
I4 = − + I2
4 4
sin x cos x 1
I2 = − + I0
2 2

But I0 = sin0 xdx = x + C1 . Therefore, back substitution yields:

sin3 x cos x 3 sin x cos x 3


∴ I4 = − − + x+C
4 8 8
∫ ∫e
(c) In = n
(ln x) dx hence show that (ln x)4 dx = 9e − 24.
1

sinm+1 x cosn−1 x n − 1
2. If Im,n = sinm x cosn xdx, show that Im,n = + Im,n−2 . Hence, compute
∫ m+n m+n
cos6 xdx.

Solution ∫ ∫
m n
sin x cos xdx = sinm x cos x cosn−1 xdx

Using integration by parts, let u = cosn−1 x and dv = sinm x cos xdx. It implies that du =
sinm+1 x
−(n − 1) cosn−2 x sin xdx and v = . Therefore,
m+1
∫ ∫
sinm x cosn xdx = uv − vdu

sinm+1 x sinm+1 x
= cos n−1
x − [−(n − 1) cosn−2 x sin xdx]
m+1 m+1

sinm+1 x cosn−1 x n−1
= + sinm+1 x cosn−2 x sin xdx
m+1 m+1

sinm+1 x cosn−1 x n−1
= + sinm x cosn−2 x sin2 xdx
m+1 m+1

sinm+1 x cosn−1 x n−1
= + sinm x cosn−2 x[1 − cos2 x]dx
m+1 m+1
[∫ ∫ ]
sinm+1 x cosn−1 x n−1
= + m
sin x cos n−2
xdx − m n
sin x cos xdx
m+1 m+1
∫ ∫
sinm+1 x cosn−1 x n−1 n−1
= + m
sin x cos n−2
xdx − sinm x cosn xdx
m+1 m+1 m+1

44
6.2 Reduction formulae ⃝Francis
c Oketch

∫ ∫ ∫
n−1 sinm+1 x cosn−1 x n − 1
⇒ m
sin x cos xdx+ n m n
sin x cos xdx = + sinm x cosn−2 xdx
m+1 m+1 m+1
∫ ∫
m+n sinm+1 x cosn−1 x n−1
⇒ m
sin x cos xdx = n
+ sinm x cosn−2 xdx
m+1 m+1 m+1
∫ ∫
sinm+1 x cosn−1 x n−1
⇒ sin x cos xdx = m n
+ sinm x cosn−2 xdx, m + n ̸= 0
m+n m+n
This can be written as

. x + n−1 I
sinm+1 x cosn−1
Im,n = m,n−2 − − − (∗)
m+n m+n

We now wish to solve cos6 xdx = I0,6 using the formula (∗) as follows:

sin x cos5 x 5
I0,6 = + I0,4
6 6
3
sin x cos x 3
I0,4 = + I0,2
4 4
sin x cos x
I0,2 = + I0,0
∫ 2 ∫
I0,0 = sin0 x cos0 xdx = dx = x + C

Back substitution yields


sin x cos x
I0,2 = +x+C
2
sin x cos3 x 3 3
I0,4 = + sin x cos x + x + C1
4 8 4
1 5 5 3 5 5
I0,6 = sin x cos x + sin x cos x + sin x cos x + x + C2
6 24 16 8

1 5 5 5
Therefore, cos6 xdx = sin x cos5 x + sin x cos3 x + sin x cos x + x + C2 , where C2 is a
6 24 16 8
constant of integration.

Exercise:
∫ ∫
n
1. [Assignment 1 ] Show that the reduction formula for x cos xdx is xn cos xdx = xn sin x +

nxn−1 cos x − n(n − 1) xn−2 cos xdx.

2. Show that the reduction formula for In = (ln x)n dx is In = x(ln x)n − nIn−1 . Hence, show
∫e
that (ln x)4 dx = 9e − 24.
1

cos2n x sin2m+1 x 2n
3. If Im,n = sin2m x cos2n+1 xdx, show that Im,n = + [Im,n−1 − Im,n ].
2m + 1 2m + 1

Lecture 8

45
6.3 Double integrals ⃝Francis
c Oketch

6.3 Double integrals


In double integrals, the focus is on a repeated process of integration of a function of two independent
variables. The double integral of a function f (x, y) over a region R is denoted by
∫∫
f (x, y)dA
R

The integral involves two dimensions which means that it deals with areas.

→ Note: in double integrals, when integrating with respect to one variable the other variable is held
constant.

Example(s):
1. Evaluate the following iterated integrals.
∫3 ∫4
(a) xydydx
0 0
Solution
∫3 ∫4 ∫3 [ ]4 ∫3 [ ]3
xy 2 8x2
xydydx = dx = 8xdx = = 36
2 2
x=0 y=0 x=0 0 x=0 0

∫1 ∫3x
(b) ey−x dydx
0 2x

Solution
∫1 ∫3x ∫1 [ ]3x ∫1 [ ] [1 ]1 (1 1)
e y−x
dydx = e y−x
dx = e2x − ex dx = e2x − ex = e2 − e +
2x 2 0 2 2
0 2x 0 0

∫1 ∫ y
(c) 2xydxdy
0 y

Solution

∫1 ∫ y ∫1 [ x2 ]√y ∫1 [y ∫1 [
y2 ] ]
2xydxdy = 2y dy = 2y − dx = y 2 − y 3 dy
2 y 2 2
0 y 0 0 0
[ y3 y 4 ]1 (1 1) 1
= − = − =
3 4 0 3 4 12

Exercise:
1. Evaluate
∫2 ∫x2
27
(a) 2xydydx [ans: = ]
4
1 x

π/2∫x2
π π2
(b) sin(y/x)dydx [ans: = 1 − + ]
2 8
0 0
∫π cos
∫ θ
1
(c) r sin θdrdθ [ans: = ]
3
0 0

46
6.3 Double integrals ⃝Francis
c Oketch

6.3.1 Finding limits of integration


When evaluating double integrals it is very common not to be told the limits of integration but simply
told that the integral is to be taken over a certain specified region R in the xy-plane. In this case, you
need to work out the limits of integration for yourself. Consider evaluating the double integral
∫∫
f (x, y)dA,
R

where R is the region enclosed by the curves y = f (x) and y = g(x) for f (x) > g(x), and dA = dydx
or dA = dxdy, in the xy-plane. The integration can be done in two ways:

(i) integrating first with respect to y and then with respect to x (i.e., dA = dydx), or

(ii) integrating first with respect to x and then with respect to y (i.e., dA = dxdy).

The limits of integration in the two approaches will in general be quite different, but both
approaches must yield the same answer.

Consider integrating first with respect to y and then with respect to x (i.e., dA = dydx). The following
procedure is used to find the limits of integration.

(i) Sketch the region of integration and label the bounding curves.

(ii) Find the y-limits of integration. Imagine a vertical line L cutting through the region R in the
direction of increasing y. Mark the y values where L enters and leaves. These give the y-limits
of integration and are usually functions of x (instead of constants).

(iii) Find the x-limits of integration. Choose x-limits that include all the vertical lines through R as
follows.

So, we can write:


R = {(x, y) : a ≤ x ≤ b, g(x) ≤ y ≤ f (x)}.
Hence, the integral becomes

∫∫ ∫b f∫(x)

f (x, y)dA = . f (x, y)dydx


R x=a y=g(x)

→ Note: to evaluate the same integral with the order of integration reversed (i.e., dA = dxdy), use
horizontal lines instead of vertical lines.

Example(s):
∫∫
1. Evaluate (y − x)dA, where R is the region bounded by the parabola y = x2 and the line
R
y = x + 2.

Solution

47
6.3 Double integrals ⃝Francis
c Oketch

The vertical line L (in the direction of the


increasing y) enters the region R at the curve
y = x2 and leaves at the line y = x + 2. The
curve and the line intersect when x = −1 and
x = 2. {So, we can write: }
R = (x, y) : −1 ≤ x ≤ 2, x2 ≤ y ≤ x + 2 .
Hence,

∫∫ ∫2 ∫
x+2 ∫2 [ ]x+2
y2
(y − x)dA = (y − x)dydx = − xy dx
2 x2
R x=−1 y=x2 x=−1
∫2 [ ] [ ]2
x4 x2 x5 x4 x3 99
= − + x3 − + 2 dx = − + − + 2x =
2 2 10 4 6 −1
20
x=−1

Exercise:
∫∫
1. Evaluate (x+2y)dA, where R is the region bounded by the parabolas y = 2x2 and y = x2 +1.
R
32
[ans: I = ]
15
∫∫
2. Evaluate xy 2 dA, where R is the triangular region bounded by the lines y = x, y = 0 and
R
1
x + y = 2. [ans: = ]
6

6.3.2 Changing the order of integration


It can happen that one order of integration is easier to evaluate than the other. In such situations,
reversing the order of integration can be useful. To change the order of integration in a double integral,
follow these steps:

1) Sketch the region of integration.

2) Set up the region according to the new order and determine the new limits of integration, one
at a time, starting with the inner variable.

3) Evaluate the new integral.

Example(s):
∫1 ∫1
1. Evaluate the iterated integral sin(y 2 )dydx by first changing the order of integration.
0 x

Solution

48
6.3 Double integrals ⃝Francis
c Oketch

∫1 ∫1 ∫1 ∫y
2
sin(y )dydx = sin(y 2 )dxdy
0 x y=0 x=0
∫1 [ ]y ∫1
= sin(y 2 ) x dy = y sin(y 2 )dy
x=0
0 0
∫1
1
= sin udu, (where u = y 2 ⇒ du = 2ydy)
2
0
[ ]1 1
= − cos u = (1 − cos 1)
0 2

∫∞ ∫∞
2e−y dydx by first changing the order of integration.[hint: sketch the region and
2
2. Evaluate
0 x
draw L parallel to x-axis (towards the increasing x)] [ans: =1]

Solution
∫∞ ∫∞ ∫∞ ∫y ∫∞ [ ]y ∫∞
−y 2 −y 2 −y 2
2ye−y dy
2
2e dydx = 2e dxdy = 2e x dy =
x=0
0 y=x 0 x=0 0 0
∫∞
−u
= e du, (where u = y 2 ⇒ du = 2ydy)
0
[ ]∞
= − e−u =1
0

Exercise:
Evaluate the following double integrals by first changing the order of integration.
√ √
∫ π∫ π
(a) cos(x2 )dxdy.
0 y

∫3 ∫9
3
(b) x3 ey dydx.
0 x2

∫1 ∫1
(c) sin(πy 2 )dydx.
0 x

Lecture 9

6.3.3 Change of variables for a double integral


The reason for changing variables is to convert the region into a nicer region to work with. We call
the equations that define the change of variables a transformation. Also, we will typically start out
with a region, R, in xy-plane and transform it into a region in uv-plane.

49
6.3 Double integrals ⃝Francis
c Oketch

Suppose that we want to integrate f (x, y) over the region R. Under the transformation x = g(u, v)
and y = h(u, v), the region becomes S and the integral becomes
∫∫ ∫∫
∂(x, y)
f (x, y)dA = . v), h(u, v))
f (g(u, dA
∂(u, v) ,
R S


∂x ∂x
∂(x, y) ∂(x, y) ∂u ∂v
where is the Jacobian of x, y with respect to u, v and is defined by = ∂y ∂y . A
∂(u, v) ∂(u, v) ∂u ∂v
transformation is valid if the Jacobian is non-zero. The Jacobian has the property that
[ ]−1
∂(x, y) ∂(u, v)
=
∂(u, v) ∂(x, y)

→ Note: here, dA is in terms of dx and dy while dA is in terms of du and dv, when converting to
single integrals. Thus, if we look just at the differentials in the above formula, it is clear that

∂(x, y)

dA = dA.
∂(u, v)

Example(s):
∫∫
1. Using the transformation u = x+2y and v = x−2y, evaluate the double integral (3x+6y)2 dA,
R
where R is the region bounded by the lines x + 2y = 2, x − 2y = 2, x + 2y = −2, and x − 2y = −2
in the xy−plane. (sketch the two regions).

Solution
Solving the equations u = x + 2y and v = x − 2y simultaneously for x and y yields
1 1
x = (u + v) and y = (u − v) − − − (∗)
2 4
Transforming each of the original equations, we get

x + 2y = 2 ⇒ u=2
x − 2y = 2 ⇒ v=2
x + 2y = −2 ⇒ u = −2
x − 2y = −2 ⇒ v = −2

Therefore, the new region is defined by S = {(u, v) : −2 ≤ u ≤ 2, −2 ≤ v ≤ 2}. The following


are the sketch of the two regions:

Now, the Jacobian for this transformation is



∂(x, y) ∂u
∂x ∂x
∂v
1

1
1
= ∂y ∂y = 21 1 = −
2
∂(u, v) ∂u ∂v
4 −4 4

50
6.3 Double integrals ⃝Francis
c Oketch

Hence, using the transformation (∗), the given integral becomes


∫∫ ∫∫ ∫2 ∫2

2 ∂(x, y)
1
2
(3x + 6y) dA = 9u dA = 9 u − dvdu
2
∂(u, v) 4
R S u=−2 v=−2
∫2 [ ]2 ∫2 [ ] [ ]2
9
= u2 v du = 9 u du = 3 u3
2
= 48
4 −2 −2
u=−2 u=−2

→ Note: the vertical line L (in the direction of the increasing v) enters the region S at the line
v = −2 and leaves at the line v = 2.
y
∫4 2∫+1
2x − y y 2x − y
(b) Using the transformation u = and v = , evaluate the double integral dxdy
2 2 y
2
0
2
(sketch the two regions).

Solution
y y
The region R is bounded by the lines x = , x = + 1, y = 0, and y = 4 in the xy−plane.
2 2
2x − y y
Solving the equations u = and v = simultaneously for x and y yields
2 2
x = u + v and y = 2v − − − (∗)
Transforming each of the original equations, we get
y 2v
x= ⇒ u+v= ⇒ u=0
2 2
y 2v
x= +1 ⇒ u+v= +1 ⇒ u=1
2 2
y = 0 ⇒ 2v = 0 ⇒ v = 0
y=4 ⇒ 2v = 4 ⇒ v=2
Therefore, the new region is defined by S = {(u, v) : 0 ≤ u ≤ 1, 0 ≤ v ≤ 2}. The following are
the sketch of the two regions:

Now, the Jacobian for this transformation is



∂(x, y) ∂u
∂x ∂x
∂v
1 1

= ∂y ∂y = =2
∂(u, v) ∂u ∂v
0 2

Hence, using the transformation (∗), the given integral becomes


∫∫ ∫∫ ∫2 ∫1 ∫2 ∫1
2x − y ∂(x, y)
dA =
u dA = u |2| dudv = 2ududv
2 ∂(u, v)
R S v=0 u=0 0 0
∫2 [ ]1 ∫2 [ ]2
= u2 dv = dv = v =2
0 0
0 0

51
6.4 Applications of double integrals ⃝Francis
c Oketch

Exercise:
the region bounded by the lines y = −2x + 4, y = −2x + 7, y = x − 2, and y = x + 1.
(a) Let R be ∫∫
Evaluate (2x2 − xy − y 2 )dxdy by applying the transformation u = x − y, v = 2x + y and
R
33
integrating over an appropriate region in the uv−plane. [ans: = ]
4
∫∫
(b) Consider the double integral (x2 − y 2 )dxdy, where D is the region bounded by the lines
D
y = x, y = x + 1, y = x−1 and y = 2x−1 . Evaluate this integral with the change of variables
u = y − x, v = xy. (sketch the two regions). [ans: =-1/2]
(c) Show that when changing to polar coordinates we have dA = rdrdθ. [hint: use the
transformation x = r cos θ, y = r sin θ]
∫∫
(d) Evaluate (x + y)dA, where R is the trapezoidal region with vertices given by (0,0), (5,0),
R
125
(2.5,2.5) and (2.5,-2.5), using the transformation x = 2u + 3v and y = 2u − 3v. [ans: = ]
4
∫∫
(e) Evaluate (x2 − xy + y 2 )dA, where R is the ellipse given by x2 − xy + y 2 ≤ 2 and using the
R
√ √ √ √ 4π
transformation x = 2u − 23 v, y = 2u + 23 v. [ans: = √ ]
3

6.4 Applications of double integrals


6.4.1 Mass, moments, and centre of mass of a physical object
The centre of mass of a physical object is its “balancing point”. Consider finding the centre of mass
of an object whose density function is ρ(x, y). Suppose R represents the shape of a thin (flat) plate,
its density is expressed as mass per unit area i.e.,
dM
ρ(x, y) =
dA
The density is a continuous function on R in the xy-plane. Thus, we use the following formulae:
∫∫
Total mass: M = ρ(x, y)dA
R
∫∫
First moment about the x-axis: Mx = yρ(x, y)dA
R
∫∫
First moment about the y-axis: My = xρ(x, y)dA
R
My
x-coordinate of centre of mass: x̄ =
M
Mx
y-coordinate of centre of mass: ȳ =
M
∴ Centre of mass: (x̄, ȳ)
→ A body’s first moments tell us about balance and about the torque the body exerts about different
axes in a gravitational field.

Example(s):
1. Find the centre of mass of a lamina defined by a triangle with vertices (0,0), (0,a), and (a,0)
with density defined as ρ(x, y) = x + y. (Sketch the region)

Solution

52
6.4 Applications of double integrals ⃝Francis
c Oketch

Equation of the line passing through points (0,a)


and (a,0) is y = a−x. The vertical line L (in the
direction of the increasing y) enters the region R
at the line y = 0 and leaves at the line y = a−x.

∫∫ ∫a ∫
(a−x) ∫a [ ](a−x)
y2 a3
Mass: M = ρ(x, y)dA = (x + y)dydx = xy + dx =
2 3
R x=0 y=0 x=0 y=0

∫∫ ∫a ∫
(a−x)
a4
First moment about the x-axis: Mx = yρ(x, y)dA = (xy + y 2 )dydx =
8
R x=0 y=0

∫∫ ∫a ∫
(a−x)
a4
First moment about the y-axis: My = xρ(x, y)dA = (x2 + xy)dydx =
8
R x=0 y=0
My 3a
x-coordinate: x̄ = =
M 8
Mx 3a
y-coordinate: ȳ = =
M 8
( 3a 3a )
∴ Centre of mass: ,
8 8
2. Find the mass, moment about the y-axis, and the x-coordinate of the centre of mass for a
triangular plate with vertices (0,0), (1,0), and (1,2) with a constant density ρ = 3g/cm2 . (Sketch
the region)

Solution

Equation of the line passing through points (0,0)


and (1,2) is y = 2x. The vertical line L (in the
direction of the increasing y) enters the region
R at the line y = 0 and leaves at the line y = 2x.

∫∫ ∫1 ∫2x
Mass: M = ρ(x, y)dA = 3dydx = 3g
R x=0 y=0
∫∫ ∫1 ∫2x
First moment about the y-axis: My = xρ(x, y)dA = 3xdydx = 2gcm
R x=0 y=0
My 2
x-coordinate: x̄ = = cm
M 3

53
6.4 Applications of double integrals ⃝Francis
c Oketch

Exercise:

1. Find the mass and center of mass of a triangular lamina with vertices (0,0), (1,0) and (0,2) if
the density function is ρ(x, y) = 1 + 3x + y. [ans: M = 8/3, center is ( 38 , 11
16 )]

2. The density at any point on a semicircular lamina is proportional to the distance from the center
of the circle of radius a. Find the center√of mass of the lamina. (place the lamina as the upper
half of the circle) [hint: ρ(x, y) = k x2 + y 2 where k is some constant, ans: M = kπa3 /3,
3a
center is (0, 2π )]

6.4.2 Centroid of geometric figures


When the density is constant, the location of the centre of mass is a feature of the geometry of the
object and not of the material from which it is made (i.e., the object is said to be homogeneous). In
such cases, the centre of mass is called the the center of gravity or the centroid. To find the centroid
of a geometric shape, set ρ = 1 and proceed to find the centre of mass as before.

Example(s):

1. Find the centroid of a homogeneous lamina which occupies the region lying between the line
y = x and the parabola y = x2 .

Solution

The vertical line L (in the direction of the


increasing y) enters the region R at the curve
y = x2 and leaves at the line y = x. The curve
and the line intersect when x = 0 and x = 1
(since at the point of intersection, x2 = x). To
determine the centroid, we set ρ = 1. So,

∫∫ ∫1 ∫x
1
Mass: M = ρ(x, y)dA = 1dydx =
6
R x=0 y=x2
∫∫ ∫1 ∫x
1
First moment about the x-axis: Mx = yρ(x, y)dA = ydydx =
15
R x=0 y=x2
∫∫ ∫1 ∫x
1
First moment about the y-axis: My = xρ(x, y)dA = xdydx =
12
R x=0 y=x2
My 1
x-coordinate: x̄ = =
M 2
Mx 2
y-coordinate: ȳ = =
M 5
(1 2)
∴ Centroid: ,
2 5

2. (a) Find the centroid of the triangular region cut from the first quadrant by the line x + y = 3.
(Sketch the region) [ans: (x̄, ȳ) = (1, 1)]

54
6.4 Applications of double integrals ⃝Francis
c Oketch

(b) Find the centroid of the triangular region cut from the first quadrant by the line y + 2x = 6.
(Sketch the region) [ans: (x̄, ȳ) = (1, 2)]

3. Find the centroid of a thin plate covering the region bounded by the parabola x2 = 4y and the
line y = 1. (Sketch the region) [ans: (x̄, ȳ) = (0, 3/5)]

6.4.3 Moments of inertia (second moments)


If a body is a rotating shaft, we are more likely to be interested in how much energy is stored in the
shaft or about how much energy it will take to accelerate the shaft to a particular angular velocity.
This is where the second moments or moments of inertia come in. Consider a body in the xy-plane,
then its moments of inertia are defined by
∫∫
Second moment about the x-axis: Ix = y 2 ρ(x, y)dA
R
∫∫
Second moment about the y-axis: Iy = x2 ρ(x, y)dA
R
Second moment about the origin: I0 = Ix + Iy (perpendicular axis theorem)

In general, the moment of inertia plays much the same role in rotational motion that mass plays in
linear motion. The moment of inertia of a wheel is what makes it difficult to start or stop the rotation
of the wheel, just as the mass of a car is what makes it difficult to start or stop the motion of the car.

Example(s):

1. A thin plate covers the triangular region bounded by the x-axis and the lines x = 1 and y = 2x
in the first quadrant. The plate’s density is given by ρ(x, y) = 6x + 6y + 6. Find the plate’s
centre of mass about the co-ordinate axes and the moments of inertia. (Sketch the region)

Solution

The vertical line L (in the direction of the


increasing y) enters the region R at the line
y = 0 and leaves at the line y = 2x.

55
6.4 Applications of double integrals ⃝Francis
c Oketch

∫∫ ∫1 ∫2x
Mass: M = ρ(x, y)dA = (6x + 6y + 6)dydx = 14
R x=0 y=0
∫∫ ∫1 ∫2x
First moment about the x-axis: Mx = yρ(x, y)dA = y(6x2 + 6y + 6)dydx = 11
R x=0 y=0
∫∫ ∫1 ∫2x
First moment about the y-axis: My = xρ(x, y)dA = x(6x2 + 6y + 6)dydx = 10
R x=0 y=0
My 5
x-coordinate: x̄ = =
M 7
Mx 11
y-coordinate: ȳ = =
M 14
( 5 11 )
∴ Centre of mass: ,
7 14

∫∫ ∫1 ∫2x
2
Moment of inertia about the x-axis: Ix = y ρ(x, y)dA = y 2 (6x + 6y + 6)dydx = 12
R x=0 y=0
∫∫ ∫1 ∫2x
2 39
Moment of inertia about the y-axis: Iy = x ρ(x, y)dA = x2 (6x + 6y + 6)dydx =
5
R x=0 y=0
39 99
Moment of inertia about the origin: Iz = Ix + Iy = 12 + =
5 5

Exercise:

1. Find the moments of inertia Ix , Iy , and I0 of a homogeneous disk D with density ρ(x, y) = ρ,
center the origin, and radius a. [ans: Ix = Iy = πρa4 /4 and I0 = πρa4 /2]

56
6.5 Triple integrals in Cylindrical and Spherical co-ordinates ⃝Francis
c Oketch

Lecture 10

6.5 Triple integrals in Cylindrical and Spherical co-ordinates


Consider the following diagram

6.5.1 Integration in Cylindrical co-ordinates


Cylindrical coordinates represent a point P in space by ordered triple (r, θ, z). The values of x, y, z, r,
and θ in Cartesian and cylindrical coordinates are related by the equations:
y
x = r cos θ, y = r sin θ, z = z, r2 = x2 + y 2 , tan θ =
x
The triple integral of a function f over the region R is obtained by
∫∫∫
I= f dV,
R

where
∂x ∂x ∂x

∂r ∂θ ∂z

∂(x, y, z)
∂y ∂y ∂y
dV = drdθdz = drdθdz
∂(r, θ, z) ∂r
∂z ∂θ ∂z
∂z ∂z

∂r ∂θ ∂z
Now,
∂x ∂x ∂x ∂y ∂y ∂y ∂z ∂z ∂z
= cos θ, = −r sin θ, = 0, = sin θ, = r cos θ, = 0, = 0, = 0, =1
∂r ∂θ ∂z ∂r ∂θ ∂z ∂r ∂θ ∂z
Hence,
cos θ −r sin θ 0


dV = sin θ r cos θ 0 drdθdz = 1(r cos2 θ + r sin2 θ)drdθdz = rdrdθdz

0 0 1
Therefore,
∫∫∫
I= f (r,.θ, z)rdrdθdz
R

∂(x, y, z)
The quantity is called the Jacobian of x, y, z with respect to r, θ, z.
∂(r, θ, z)
Example(s):

57
6.5 Triple integrals in Cylindrical and Spherical co-ordinates ⃝Francis
c Oketch

1. Find the limits of integration in Cylindrical co-ordinates for integrating a function f (r, θ, z) over
the region R bounded below by the plane z = 0, laterally by the circular cylinder x2 +(y−1)2 = 1
and above by the paraboloid z = x2 + y 2 .

Solution
∫∫∫
To evaluate f (r, θ, z)dV , we need to integrate first with respect to z, then with respect to
R
r, and with respect to θ. The following steps are necessary.

i) Sketch the region R along with its projection D on the xy-plane. Label the surfaces and
curves that bound R and D.

Here, the base R is also the region’s projection D on the xy-plane. The boundary of D is
the circle x2 + (y − 1)2 = 1 whose polar coordinate equation is r2 − 2r sin θ = 0.
ii) We find the limits of integration starting with the z-limits. A line M (towards the increasing
z) through a typical point (r, θ) in D parallel to the z-axis enters R at z = 0 and leaves at
z = x2 + y 2 = r 2 .
iii) Next, we find the r limits of integration. A ray L through (r, θ) from the origin enters R
at r = 0 and leaves at r = 2 sin θ.
iv) Finally, we find the θ limits of integration. As L sweeps through D, the angle θ it makes
with the positive x-axis runs from θ = 0 to θ = π.

Thus the integral is

∫∫∫ ∫π 2∫sin θ ∫r2


f (r, θ, z)dV = f (r, θ, z)dzrdrdθ
R θ=0 r=0 z=0

2. Find the centroid of the solid enclosed by the cylinder x2 + y 2 = 4 bounded above by the
paraboloid z = x2 + y 2 and bounded below the xy-plane.

Solution

The solid’s centroid (x̄, ȳ, z̄) lies on the axis of symmetry (the z-axis). Thus, x̄ = ȳ = 0. So,

58
6.5 Triple integrals in Cylindrical and Spherical co-ordinates ⃝Francis
c Oketch

Mxy
z̄ = . For centroid, we set ρ(r, θ, z) = 1. Hence,
M
∫∫∫ ∫2π ∫2 ∫r2
M= ρ(r, θ, z)dV = 1dzrdrdθ = 8π
R θ=0 r=0 z=0
∫∫∫ ∫2π ∫2 ∫r2
32π
Mxy = zρ(r, θ, z)dV = zdzrdrdθ =
3
R θ=0 r=0 z=0
Mxy 4
∴ z̄ = =
M 3
Thus, the centroid is (0, 0, 43 ).

6.5.2 Integration in Spherical co-ordinates


Spherical coordinates represent a point P in space by ordered triple (ρ, ϕ, θ). The values of x, y, z, r,
and ρ in Cartesian and Spherical coordinates are related by the equations:

x = ρ sin ϕ cos θ, y = ρ sin ϕ sin θ, z = ρ cos ϕ, r = ρ sin ϕ, ρ= r2 + z 2 , where ρ ≥ 0, 0 ≤ ϕ ≤ π

Exercise:

1. Find the spherical co-ordinate equation for the sphere x2 + (y − 1)2 + z 2 = 1

Solution
Substituting the values of x, y and z in the given equation yields

x2 + (y − 1)2 + z 2 = 1 ⇒ (ρ sin ϕ cos θ)2 + [ρ sin ϕ sin θ − 1]2 + (ρ cos ϕ)2 = 1


⇒ ρ2 sin2 ϕ cos2 θ + ρ2 sin2 ϕ sin2 θ − 2ρ sin ϕ sin θ + 1 + ρ2 cos2 ϕ = 1
⇒ ρ2 sin2 ϕ[cos2 θ + sin2 θ] + ρ2 cos2 ϕ = 2ρ sin ϕ sin θ ⇒ ρ2 sin2 ϕ + ρ2 cos2 ϕ = 2ρ sin ϕ sin θ
⇒ ρ2 [sin2 ϕ + cos2 ϕ] = 2ρ sin ϕ sin θ ⇒ ρ2 = 2ρ sin ϕ sin θ ⇒ ρ = 2 sin ϕ sin θ

On maps of the earth, θ is related to the meridian of a point on the earth and ϕ is its latitude, while ρ
is related to the elevation above the earth’s surface. The triple integral of a function f over the region
R is obtained by ∫∫∫
I= f dV,
R

where
∂x ∂x ∂x

∂ρ ∂ϕ ∂θ

∂(x, y, z) ∂y
dV = dρdϕdθ = ∂y ∂y
dρdϕdθ
∂ρ ∂θ
∂(ρ, ϕ, θ) ∂ϕ
∂z ∂z ∂z

∂ρ ∂ϕ ∂θ
Now,
∂x ∂x ∂x ∂y ∂y
= sin ϕ cos θ, = ρ cos ϕ cos θ, = −ρ sin ϕ sin θ, = sin ϕ sin θ, = ρ cos ϕ sin θ,
∂ρ ∂ϕ ∂θ ∂ρ ∂ϕ
∂y ∂z ∂z ∂z
= ρ sin ϕ cos θ, = cos ϕ, = −ρ sin ϕ, =0
∂θ ∂ρ ∂ϕ ∂θ

59
6.5 Triple integrals in Cylindrical and Spherical co-ordinates ⃝Francis
c Oketch

Hence,

sin ϕ cos θ ρ cos ϕ cos θ −ρ sin ϕ sin θ

∂(x, y, z)
= sin ϕ sin θ ρ cos ϕ sin θ ρ sin ϕ cos θ
∂(ρ, ϕ, θ)
cos ϕ −ρ sin ϕ 0
[ ] [ ]
= cos ϕ ρ2 cos ϕ sin ϕ cos2 θ + ρ2 cos ϕ sin ϕ sin2 θ + ρ sin ϕ ρ sin2 ϕ cos2 θ + ρ sin2 ϕ sin2 θ
[ ] [ ]
= cos ϕρ2 cos ϕ sin ϕ cos2 θ + sin2 θ + ρ sin ϕρ sin2 ϕ cos2 θ + sin2 θ
[ ]
= ρ2 cos2 ϕ sin ϕ + ρ2 sin ϕ sin2 ϕ = ρ2 sin ϕ cos2 ϕ + sin2 ϕ = ρ2 sin ϕ

Therefore,
∫∫∫
2
I= f (ρ, ϕ, θ)ρ
. sin ϕdρdϕdθ
R

∂(x, y, z)
The quantity is called the Jacobian of x, y, z with respect to ρ, ϕ, θ.
∂(ρ, ϕ, θ)
Example(s):
π
1. Find the volume of the “icecream cone” R cut from the solid sphere ρ ≤ 1 by the cone ϕ = .
3

Solution

∫∫∫
Volume is V = ρ2 sin ϕdρdϕdθ since the integral of f (ρ, ϕ, θ) = 1 over R.
R

ρ-limits: the ray M enters R at ρ = 0 and leaves at ρ = 1.


π
ϕ-limits: the cone makes an angle of with the positive z-axis. For any given θ, the angle
3
π
ϕ can run from ϕ = 0 to ϕ = .
3
θ-limits: the ray L sweeps over R as θ runs from 0 to 2π.

Thus the volume is


∫2π ∫
π/3 ∫1
π
V = ρ2 sin ϕdρdϕdθ =
3
θ=0 ϕ=0 ρ=0

60
⃝Francis
c Oketch

Lecture 11

7 Matrices
7.1 Linear systems
A linear system of n algebraic equations in n unknowns x1 , x2 , · · · , xn is a set of equations of the form

a11 x1 + a12 x2 + · · · + a1n xn = b1


a21 x1 + a22 x2 + · · · + a2n xn = b2
.. .. . . .. .. (11)
. . . . .
an1 x1 + an2 x2 + · · · + ann xn = bn

where the coefficients aij and bi are known numbers. Using matrix notation, we can write equation
(11) as
AX⃗ = B,

 
a11 a12 · · · a1n

 a21 a22 · · · a2n 

where the coefficient matrix A = [aij ] is the n × n matrix given by A = 
 .. .. .. .. 
 and
 . . . . 
an1 an2 · · · ann
   
x1 b1
   
x  b 
⃗ =  .2 , B
X ⃗ =  .2 .
 .  .
 .  .
xn bn

7.2 Determinants

Example(s):
 
2 3 −1
 
1. Find the determinant of the matrix A = 4 1 −3.
3 −2 5

Solution

2 3 −1
1 −3 4 −3 4 1

det(A) = |A| = 4 1 −3 = 2 − 3 + (−1)
−2 5 3 5 3 −2
3 −2 5
= 2(5 − 6) − 3(20 + 9) − 1(−8 − 3)
= −2 − 87 + 11 = −78

→ Note: a matrix is said to be singular if its determinant is zero. Thus, a singular matrix has no
inverse.

7.3 Matrix inverse


The inverse of a non-singular square matrix A = [aij ] of size n × n is given by

1
A−1 = . adj(A) ,
|A|

where the adjoint matrix of A, denoted by adj(A), is given as

adj(A) = [coef(A)]T .

61
7.3 Matrix inverse ⃝Francis
c Oketch

The superscript T denotes the matrix transpose and is obtained by interchanging the corresponding
rows and columns. The cofactor matrix of A, i.e., coef(A) = [cij ] is the n × n matrix given as
 
c11 c12 · · · c1n

 c21 c22 · · · c2n 

coef(A) = 
 .. .. .. .. 

 . . . . 
cn1 cn2 · · · cnn

The elements (or cofactors) cij of aij , for i, j = 1, 2, · · · , n, are given by the formula:


cij = (−1). i+j Mij

The matrix Mij is an (n − 1) × (n − 1) submatrix of A and is obtained by deleting the ith row and

the j th column of the matrix A. The determinant, i.e., Mij , is called the minor of aij .

Example(s):
 
1 3 2
 
1. Find the inverse of the matrix A =  0 5 1.
−1 3 0

Solution
The inverse of the matrix A is given by the formula
1
A−1 = adj(A),
|A|

where adj(A) = [coef(A)]T . Since A is a 3 × 3 matrix, the cofactor matrix, coef(A), takes the
form  
c11 c12 c13
 
coef(A) = c21 c22 c23 
c31 c32 c33
The cofactors are given as:

5 1

c11 = (−1)1+1 = (−1)2 (0 − 3) = −3
3 0

1
1+2 0
c12 = (−1) = (−1)3 (0 − −1) = −1
−1 0

0 5

c13 = (−1)1+3 = (−1)4 (0 − −5) = 5
−1 3


2+1 3 2
c21 = (−1) = (−1)3 (0 − 6) = 6
3 0

1 2

c22 = (−1)2+2 = (−1)4 (0 − −2) = 2
−1 0

3
2+3 1
c23 = (−1) = (−1)3 (3 − −3) = −6
−1 3


3+1 3 2
c31 = (−1) = (−1)4 (3 − 10) = −7
5 1


3+2 1 2
c32 = (−1) = (−1)5 (1 − 0) = −1
0 1


3+3 1 3
c33 = (−1) = (−1)6 (5 − −0) = 5
0 5

62
7.3 Matrix inverse ⃝Francis
c Oketch

Therefore, the cofactor matrix of A is


 
−3 −1 5
 
coef(A) =  6 2 −6
−7 −1 5

Hence, the adjoint matrix of A is given as


 
−3 6 −7
T  
adj(A) = [coef(A)] == −1 2 −1 .
5 −6 5

The determinant of the matrix A is given as:



1 3 2
5 1 0 1 0 5

|A| = 0 5 1 = 1 − 3 + 2
3 0 −1 0 −1 3
−1 3 0
= 1(0 − 3) − 3(0 + 1) + 2(0 + 5) = 4

Therefore,  
−3 6 −7
1 
A−1 = −1 2 −1 .
4
5 −6 5

2. Solve the following system using matrix inverse method.

3x − y + 2z = 1
x+y+z =4
2x − y + z = −1

Solution
In matrix form, we have     
3 −1 2 x 1
    
1 1 1 y  =  4 
2 −1 1 z −1
Therefore,      
3 −1 2 x 1
  ⃗   ⃗  
A = 1 1 1 , X = y  and B =  4 
2 −1 1 z −1
⃗ we pre-multiply both sides by A−1 to get A−1 AX
⃗ = B,
Since AX ⃗ = A−1 B
⃗ ⇒ ⃗ = A−1 B.
X ⃗
Hence,  
x
  −1 ⃗
y  = A B,
z
1
where A−1 = adj(A) and the adjoint matrix of A is defined by adj(A) = [coef(A)]T . Since A
|A|
is a 3 × 3 matrix, the cofactor matrix, coef(A), takes the form
 
c11 c12 c13
 
coef(A) = c21 c22 c23 
c31 c32 c33

63
7.3 Matrix inverse ⃝Francis
c Oketch

The cofactors are given as:



1 1
1+1
c11 = (−1) = (−1)2 (1 − −1) = 2
−1 1

1 1

c12 = (−1)1+2 = (−1)3 (1 − 2) = 1
2 1

1
1+3 1
c13 = (−1) = (−1)4 (−1 − 2) = −3
2 −1

−1 2

c21 = (−1)2+1 = (−1)3 (−1 − −2) = −1
−1 1


2+2 3 2
c22 = (−1) = (−1)4 (3 − 4) = −1
2 1

3 −1

c23 = (−1)2+3 = (−1)3 (−3 − −2) = 1
2 −1


3+1 −1 2
c31 = (−1) = (−1)4 (−1 − 2) = −3
1 1


3+2 3 2
c32 = (−1) = (−1)5 (3 − 2) = −1
1 1


3+3 3 −1
c33 = (−1) = (−1)6 (3 − −1) = 4
1 1

Therefore, the cofactor matrix of A is


 
2 1 −3
 
coef(A) = −1 −1 1 
−3 −1 4

Hence, the adjoint matrix of A is given as


 
2 −1 −3
 
adj(A) = [coef(A)]T =  1 −1 −1 .
−3 1 4

The determinant of the matrix A is given as:



3 −1 2
1 1 1 1 1 1

|A| = 1 1 1 = 3 + 1 + 2
−1 1 2 1 2 −1
2 −1 1
= 3(1 + 1) + 1(1 − 2) + 2(−1 − 2) = −1
   
2 −1 −3 −2 1 3
1    
Hence, A−1 =  1 −1 −1 = −1 1 1 . So,
−1
−3 1 4 3 −1 −4
      
x −2 1 3 1 −1
  −1 ⃗     
y  = A B = −1 1 1  4  =  2 
z 3 −1 −4 −1 3

Therefore, x = −1, y = 2 and z = 3.

Exercise:

1. Solve the following system of linear algebraic equations using matrix inverse method.

64
7.4 Cramer’s rule ⃝Francis
c Oketch

(a)
7x + 2y + z = 21
3y − z = 5
−3x + 4y − 2z = −1
(b)
x + 2y + 3z = 9
2x − y + z = 8
3x − z = 3

7.4 Cramer’s rule


⃗ ̸= ⃗0.
This rule is applicable for a non-homogeneous system, i.e., a system in which B

Example(s):
1. Solve the following system using Cramer’s rule.
3x − y + 2z = 1
x+y+z =4
2x − y + z = −1

Solution
In matrix form we have
      
3 −1 2 x 1 3 −1 2
      
1 1 1 y  =  4  ⇒ A = 1 1 1  ⇒ |A| = −1
2 −1 1 z −1 2 −1 1

1 −1 2


4 1 1

−1 −1 1 1(1 + 1) + 1(4 + 1) + 2(−4 + 1)
x = = = −1
|A| −1

3 1 2


1 4 1

2 −1 1 3(4 + 1) − 1(1 − 2) + 2(−1 − 8)
y = = =2
|A| −1

3 −1 1


1 1 4

2 −1 −1 3(−1 + 4) + 1(−1 − 8) + 1(−1 − 2)
z = = =3
|A| −1

Exercise:
1. Solve the following system using Cramer’s rule. [ans: x = 4, y = −2, z = 1]
2x + 3y − z = 1
4x + y − 3z = 11
3x − 2y + 5z = 21
2. Use Cramer’s rule to solve the following system.
2x + 3y − z = 2
x + 4y − 2z = −2
4x − y + z = 1

3. Use Cramer’s rule to solve the following system. [ans: x = 1, y = 3, z = −2]


2x + 3y − 5z = 1
x+y−z =2
2y − z = 8

65
7.5 Eigenvalues and eigenvectors ⃝Francis
c Oketch

7.5 Eigenvalues and eigenvectors


Consider the system
⃗ = B,
AX ⃗

where A is a square matrix, X⃗ is an unknown column vector. If B ⃗ = ⃗0, then the above system is said
⃗ ̸= ⃗0, then the above system is said to be non-homogeneous (inhomogeneous).
to be homogeneous. If B
Consider the homogeneous system
AX⃗ = ⃗0

If |A| ̸= 0 ⇒ X ⃗ = ⃗0 which is a trivial solution of the homogeneous system. For non-trivial


solution, we require that |A| = 0. Thus, there exists X ̸ ⃗0 and a parameter (or a scalar) λ such that
⃗ =

⃗ = λI X
AX ⃗ ⇒ (A − λI)X
⃗ = ⃗0,

where I is a square unit matrix of same order as A. The values of λ are called eigenvalues of matrix
A while the corresponding values of X⃗ are the eigenvectors of matrix A. To find the eigenvalues of A,
we find the roots of the characteristic equation

|A − λI|
. =0

Example(s):

1. Find  the eigenvalues


 and the corresponding (associated) eigenvectors of the matrix
7 0 −3
 
A = −9 −2 3 .
18 0 −8

Solution
   
1 0 0 λ 0 0
   
The characteristic equation is |A − λI| = 0. Now, λI = λ 0 1 0 =  0 λ 0 . Hence,
0 0 1 0 0 λ

7 − λ −3
0

|A − λI| = 0 ⇒ −9 −2 − λ 3 =0

18 0 −8 − λ

⇒ (7 − λ)[(−2 − λ)(−8 − λ) − 0] − 3[0 − 18(−2 − λ)] = 0


⇒ (7 − λ)(2 + λ)(8 + λ) − 54(2 + λ) = 0
⇒ (2+λ)[(7−λ)(8+λ)−54] = 0 ⇒ (2+λ)[2−λ−λ2 ] = 0 ⇒ (2+λ)(2+λ)(1−λ) = 0
λ = 1 or λ = −2

 −2.
Thus, the eigenvalues of the matrix A are 1 and  To get the corresponding eigenvectors, we
x1
solve the system (A − λI)X ⃗ =
⃗ = ⃗0, where X 
x 2 .
x3

Case 1: using λ = 1, the system reduces to


    
6 0 −3 x1 0 6x1 − 3x3 =0
    
−9 −3 3  x2  = 0 ⇒ −9x1 − 3x2 + 3x3 = 0
18 0 −9 x3 0 18x1 − 9x3 =0

From the first and the third equations, we have x3 = 2x1 . Substituting in the second
equation we get x2 =−x1 . Therefore,
  the eigenvector associated with (or corresponding
x1 1
⃗ =
to) λ = 1 is X
  
x2  = x1 −1, where x1 is an arbitrary constant.
x3 2

66
7.5 Eigenvalues and eigenvectors ⃝Francis
c Oketch

Case 2: using λ = −2, the system reduces to


    
9 0 −3 x1 0 9x1 − 3x3 = 0
    
−9 0 3  x2  = 0 ⇒ −9x1 + 3x3 = 0
18 0 −6 x3 0 18x1 − 6x3 = 0

From these equations, we have x


3 = 3x1 
for any
 valueofx2 . Therefore,
  the eigenvector
x1 x1 1 0
⃗ =
corresponding to λ = −2 is X
    
x2  =  x2  = x1 0 + x2 1.
 
x3 3x1 3 0

Exercise:

1. [Assignment 2 ] Determine  the eigenvalues√ and


 the eigenvector associated with the largest
1 −3 2
√ 

eigenvalue of the matrix A = √−3 1
√ − 2.
2 − 2 4
 
0 1 1
 
2. Determine the eigenvalues and the associated eigenvectors of the matrix A = 1 0 1. [ans:
1 1 0
   
1 −1
   
eigenvalues λ = −1, λ = 2, eigenvectors: λ = 2 yields x3 1 and λ = −1 yields x2  1  +
1 0
 
−1
 
x3  0 ]
1

67

You might also like